Exam 8

Pataasin ang iyong marka sa homework at exams ngayon gamit ang Quizwiz!

A clinical study is performed in which the subjects are children 1 to 4 years old who have had multiple infections with viral, fungal, and parasitic diseases. Compared with a normal control group, these children do not have a subpopulation of cells lacking surface immunoglobulin that mark with CD1a, CD2, CD3, CD4, and CD8. Which of the following karyotypic abnormalities is most likely to be seen in the children in this study? □ (A) +21 □ (B) 22q11.2 □ (C) t(9;22) □ (D) t(15;17) □ (E) X(fra) □ (F) XXY

22q11.2

A 35 year old man presents to your clinic upon returning from a six-month trip to the Peruvian Amazon. He has a 4 X 4 cm round ulcerative painless lesion on his left leg (shown below), with ipsilateral inguinal lymphadenopathy. A leading edge biopsy reveals macrophages with vacuoles full of amastigotes. In the course of this visit, you advise him as follows. (A) An Anopheles spp. mosquito is likely responsible for his infection. (B) A Simulium spp. black fly is likely responsible for his infection. (C) A Lutzomyia spp. sandfly is likely responsible for his infection. (D) Tainted water is likely responsible for his infection. (E) Unprotected sex during a trip to Carnival was responsible for his infection.

A Lutzomyia spp. sandfly is likely responsible for his infection

A 53 year old sexually-active man presents to his physician because of dull pain in the pelvis and vague discomfort when straining or with urination. He is afrebrile and his WBC is within normal limits. On rectal examination, the prostate is boggy. Fluid is expressed after prostatic massage. Culture of the expressed fluid is sterile. Which of the following is the best diagnosis? (A) Prostatic adenocarcinoma (B) Benign prostatic hyperplasia (C) Abacterial prostatitis (D) Tuberculous prostatitis (E) Acute cystitis

Abacterial prostatitis

A 60-year-old, previously healthy man sees his physician because he feels feverish and weak. He reports passing dark-colored urine on several occasions during the past month, but has no urinary frequency, dysuria, or nocturia. On physical examination, his temperature is 37.8°C, and blood pressure is 125/85 mm Hg. A dipstick urinalysis shows 4+ hematuria; 1+ proteinuria; and no glucose or ketones. Which of the following procedures is the most appropriate in management of this patient? □ (A) Straining of urine for calculi □ (B) Urine microbiologic culture □ (C) Abdominal CT scan for renal mass □ (D) Collection of a 24-hour urine specimen for protein □ (E) Percutaneous renal biopsy

Abdominal CT scan for renal mass

A 7-year-old boy has complained of worsening pain in the right side of his groin region for the past week. Physical examination shows painful, swollen lymph nodes in the right inguinal region. An inguinal lymph node biopsy is performed. Histologically, the node has large, variably sized, germinal centers containing tingible-body macrophages and numerous mitotic figures. There are numerous parafollicular and sinusoidal neutrophils. What is the most likely cause of these histologic changes? □ (A) Acute lymphoblastic leukemia □ (B) Sarcoidosis □ (C) Follicular lymphoma □ (D) Cat-scratch disease □ (E) Acute lymphadenitis □ (F) Toxoplasmosis

Acute lymphadenitis

A 4-year-old boy has appeared listless for about 1 week. He now complains of pain when he is picked up by his mother, and he exhibits irritability when his arms or legs are touched. In the past 2 days, several large ecchymoses have appeared on the right thigh and left shoulder. CBC shows hemoglobin, 10.2 g/dL; hematocrit, 30.5%; MCV, 96 μm3; platelet count, 45,000/mm3; and WBC count, 13,990/mm3. Examination of the peripheral blood smear shows blasts that lack peroxidase-positive granules, but contain PAS-positive aggregates and stain positively for TdT. Flow cytometry shows the phenotype of blasts to be CD19+, CD3−, and sIg−. Which of the following is the most likely diagnosis? □ (A) Chronic myelogenous leukemia □ (B) Idiopathic thrombocytopenic purpura □ (C) Acute myelogenous leukemia □ (D) Chronic lymphocytic leukemia □ (E) Acute lymphoblastic leukemia

Acute lymphoblastic leukemia

A 45 year old man has experienced gradual weight loss, weakness, anorexia, and easy fatigability for months. Physical examination reveals marked splenomegaly. CBC yields hemoglobin 12.9 g/dL, hematocrit 38.1%, MCV 92 fL, platelet count 410,000/mm3, and WBC count 168,000/mm3. The peripheral blood smear is depicted in figure A. Karyotypic analysis shows the Ph chromosome. He undergoes chemotherapy with imatinib, which reduces the spleen size and brings the total leukocyte count within normal range. He remains in remission for three years and then begins to experience fatigue and a 10-kg weight loss. Physical examination now reveals petechial hemorrhages. CBC yields hemoglobin 10.5 g/dL, hematocrit 30%, platelet count 60,000/ mm3, and WBC count 40,000/ mm3. A peripheral blood smear is depicted in figure B. Karyotypic analysis shows two Ph chromosomes and aneuploidy. Flow cytometric analysis of the peripheral blood shows CD19+, CD10+, sIg-, CD3- cells. Which of the following complications of the initial disease did this patient develop following therapy? (A) Sézary syndrome (B) Myelodysplastic syndrome (C) Mantle cell lymphoma (D) Chronic lymphocytic leukemia (E) Acute lymphoblastic leukemia

Acute lymphoblastic leukemia

A 38-year-old woman visits her physician because she has had bleeding gums for the past 3 weeks. Physical examination shows that her gingivae are thickened and friable. She has hepatosplenomegaly and generalized nontender lymphadenopathy. CBC shows hemoglobin, 11.2 g/dL; hematocrit, 33.9%; MCV, 89 μm 3; platelet count, 95,000/mm3; and WBC count, 4500/mm3 with 25% segmented neutrophils, 10% bands, 2% metamyelocytes, 55% lymphocytes, 8% monocytes, and 1 nucleated RBC per 100 WBCs. A bone marrow biopsy specimen shows 100% cellularity, with many large blasts that are peroxidase negative and nonspecific esterase positive. What is the most likely diagnosis? □ (A) Acute lymphoblastic leukemia □ (B) Acute megakaryocytic leukemia □ (C) Acute promyelocytic leukemia □ (D) Acute erythroleukemia □ (E) Acute monocytic leukemia

Acute monocytic leukemia

A 22-year-old university student reports easy fatigability of 2 months' duration. On physical examination, she has no hepatosplenomegaly or lymphadenopathy. Mucosal gingival hemorrhages are noted. CBC shows hemoglobin, 9.5 g/dL; hematocrit, 28.2%; MCV, 94 μm3; platelet count, 20,000/mm3; and WBC count, 107,000/mm3. A bone marrow biopsy specimen shows that the marrow is 100% cellular with few residual normal hematopoietic cells. Most of the cells in the marrow are large, with nuclei having delicate chromatin and several nucleoli. The cytoplasm of these cells has azurophilic, peroxidase-positive granules. Which of the following is the most likely diagnosis? □ (A) Acute lymphoblastic leukemia □ (B) Acute myelogenous leukemia □ (C) Chronic lymphocytic leukemia □ (D) Chronic myelogenous leukemia □ (E) Hodgkin lymphoma □ (F) Sézary syndrome

Acute myelogenous leukemia

A 60 year old woman develops fatigue and progressive shortness of breath. On physical examination, she has multiple ecchymoses on the chest, back, upper arms, and legs. Laboratory studies reveal Hct 23%, WBC count 2000/mm3 and platelet count 45,000/mm3. Cytogenetic studies are positive for t (8;21) (q22;q22). A bone marrow biopsy and aspirate smear are shown below. Which of the following is the most likely diagnosis? (A) Acute myelogenous leukemia M2 (B) Acute lymphocytic leukemia (C) Follicular lymphoma (D) Chronic myelogenous leukemia (E) Myelodysplasia

Acute myelogenous leukemia M2

A 50 year old man develops fatigue and progressive shortness of breath. On physical examination, there are ecchymoses on the chest and back. Laboratory studies reveal Hct 30%, WBC count 5000/mm3, and platelet count 105,000/mm3. A bone marrow biopsy is shown below. Cytogenetic analysis yields t (15;17) (q22;q12). Which of the following is the most likely diagnosis? (A) Acute myelogenous leukemia M1 (B) Acute promyelocytic leukemia M3 (C) Follicular lymphoma (D) Chronic myelogenous leukemia (E) Myelodysplasia

Acute promyelocytic leukemia M3

A 17-year-old boy is involved in a motor vehicle accident in which he sustains severe blunt trauma to the extremities and abdomen. Over the next 3 days, he develops oliguria and dark brown urine. The urine dipstick analysis is positive for myoglobin and for blood, but microscopic examination of the urine shows no RBCs. His serum urea nitrogen level increases to 38 mg/dL, and he undergoes hemodialysis for 3 weeks. His condition improves, but the urine output remains greater than 3 L/day for 1 week before the urea nitrogen returns to normal. Which of the following renal lesions was most likely present in this patient? □ (A) Malignant nephrosclerosis □ (B) Renal vein thrombosis □ (C) Membranous glomerulonephritis □ (D) Acute pyelonephritis □ (E) Acute tubular necrosis

Acute tubular necrosis

A 26 year old man is involved in a motor vehicle accident and sustains acute blood loss. He is hypotensive for several hours before paramedical personnel arrive. They stabilize the bleeding and transport him to a hospital, where he receives a transfusion of three units of packed RBCs. During the next week, his serum urea nitrogen increases to 48 mg/dL, his serum creatinine increases to 5 mg/dL, and urine output decreases. He undergoes hemodialysis for the next two weeks and then develops marked polyuria, with urine output of 2-3 L/day. His recovery is complicated by bronchopneumonia, but renal function gradually returns to normal. Which of the following is the most likely cause of this patient's transient renal disease? (A) Crescentic glomerulonephritis (B) Chronic pyelonephritis (C) Arterinephrosclerosis (D) Diabetic nephropathy (E) Acute tubular necrosis

Acute tubular necrosis

A 58-year-old man is in stable condition after an acute myocardial infarction. Two days later, his urine output decreases, and the serum urea nitrogen level increases to 3.3 mg/dL. Oliguria persists for 5 days, followed by polyuria for 2 days. He is discharged from the hospital. Which of the following renal lesions best explains these renal abnormalities? □ (A) Acute tubular necrosis □ (B) Benign nephrosclerosis □ (C) Acute renal infarction □ (D) Hemolytic-uremic syndrome □ (E) Rapidly progressive glomerulonephritis

Acute tubular necrosis

A 70-year-old, previously healthy man comes to his physician for a routine health examination. On palpation, his prostate is normal in size. Laboratory studies show a serum prostate-specific antigen (PSA) level of 17 ng/mL, however, twice the value he had 1 year ago. A routine urinalysis shows no abnormalities. Which of the following histologic findings in a subsequent biopsy specimen of the prostate is most likely to account for the patient's current status? □ (A) Acute prostatitis □ (B) Adenocarcinoma □ (C) Chronic abacterial prostatitis □ (D) Nodular hyperplasia □ (E) Prostatic intraepithelial neoplasia

Adenocarcinoma

A 64 year old man presents with complaints of weight loss due to anorexia, nausea, fatigue, paresthesia, and noticeable weakness. Laboratory findings include hematocrit 32%, hemoglobin 10.5 mg/dL, and MCV 125 fL. How should this patient's anemia be treated and what is the most likely cause? (A) Prescribe a folic acid supplement to correct folate deficiency. (B) Prescribe erythropoietin to treat anemia resulting from presumptive renal failure (C) Administer injections of vitamin B12 to combat pernicious anemia (D) Prescribe an iron supplement to correct an iron-poor diet (E) Because the anemia cannot be categorized from the data provided, prescribe folic acid, vitamin B12, iron, and erythropoietin until the cause is determined

Administer injections of vitamin B12 to combat pernicious anemia

A 60-year-old man has experienced vague abdominal discomfort accompanied by bloating and diarrhea for the past 6 months. On physical examination, there is a mid-abdominal firm mass. The stool is positive for occult blood. An abdominal CT scan shows a 5 × 12 cm mass involving the wall of the distal ileum and adjacent mesentery. A laparotomy is performed, and the mass is removed. Microscopically, the mass is composed of sheets of large lymphoid cells with large nuclei, prominent nucleoli, and frequent mitoses. The neoplastic cells mark with CD19+ and CD20+ and have the BCL6 gene rearrangement. Which of the following prognostic features is most applicable to this case? □ (A) Indolent, survival of 7 to 9 years without treatment □ (B) Indolent, can be cured by chemotherapy □ (C) Indolent, often undergoes spontaneous remission □ (D) Aggressive, can be cured by chemotherapy □ (E) Aggressive, often transforms to acute leukemia □ (F) Aggressive, often spreads to liver, spleen, and marrow

Aggressive, can be cured by chemotherapy

A 15-year-old boy has been passing dark-colored urine for the past month. On physical examination, he has bilateral sensorineural hearing loss and corneal erosions. Urinalysis shows a pH of 6.5; specific gravity 1.015; 1+ hematuria; 1+ proteinuria; and no ketones, glucose, or leukocytes. The serum creatinine level is 2.5 mg/dL, and the urea nitrogen level is 24 mg/dL. A renal biopsy specimen shows tubular epithelial foam cells by light microscopy. By electron microscopy, the glomerular basement membrane shows areas of attenuation, with splitting and lamination of lamina densa in other thickened areas. What is the most likely diagnosis? □ (A) Acute tubular necrosis □ (B) Berger disease □ (C) Membranous glomerulonephritis □ (D) Diabetic nephropathy □ (E) Alport syndrome

Alport syndrome

A 30-year-old, previously healthy man has had an enlarging nodular area on his arm for the past 8 months. On physical examination, there is an ulcerated, reddish violet, 3 × 7 cm lesion on his right forearm and nontender right axillary and left inguinal lymphadenopathy. A chest radiograph shows a 4-cm nodular left pleural mass. An abdominal CT scan shows a 5-cm right retroperitoneal mass. Biopsy of an inguinal node is performed, and microscopic examination shows large anaplastic cells, some of which contain horseshoe-shaped nuclei and voluminous cytoplasm. The tumor cells cluster around venules and infiltrate sinuses. The patient goes into remission after chemotherapy. Which of the following immunohistochemical markers is most likely to be positive in the tumor cells? □ (A) Anaplastic lymphoma kinase protein □ (B) CD10 □ (C) c-KIT proto-oncogene □ (D) HTLV-1 □ (E) p24 antigen

Anaplastic lymphoma kinase protein

A 50-year-old man has experienced chronic fatigue and weight loss for the past 3 months. There are no remarkable findings on physical examination. Laboratory studies show hemoglobin, 11.2 g/dL; hematocrit, 33.3%; MCV, 91 μm3; platelet count, 240,000/mm3; WBC count, 7550/mm3; serum iron 80 μg/dL; total iron-binding capacity, 145 μg/dL; and serum ferritin, 565 ng/mL. The ANA test result is positive. Which of the following is the most likely diagnosis? □ (A) Iron deficiency anemia □ (B) Aplastic anemia □ (C) Anemia of chronic disease □ (D) Microangiopathic hemolytic anemia □ (E) Megaloblastic anemia □ (F) Thalassemia minor

Anemia of chronic disease

A 33-year-old woman has had fever and increasing fatigue for the past 2 months. Over the past year, she has noticed soreness of her muscles and joints and has had a 4-kg weight loss. On physical examination, her temperature is 37.5°C, pulse is 80/min, respirations are 15/min, and blood pressure is 145/95 mm Hg. She has pain on deep inspiration, and a friction rub is heard on auscultation of the chest. Laboratory findings show glucose, 73 mg/dL; total cholesterol, 160 mg/dL; total protein, 5.2 g/dL; albumin, 2.9 g/dL; total bilirubin, 0.9 mg/dL; and creatinine, 2.4 mg/dL. Serum complement levels are decreased. CBC shows hemoglobin of 9.7 g/dL, platelet count of 85,000/mm3, and WBC count of 3560/mm3. A renal biopsy specimen shows a diffuse proliferative glomerulonephritis with extensive granular immune deposits of IgG and C1q in capillary loops and mesangium. After being treated with immunosuppressive therapy consisting of prednisone and cyclophosphamide, her condition improves. Which of the following serologic studies is most likely to be positive in this patient? □ (A) Anticentromere antibody □ (B) Anti-double-stranded DNA antibody □ (C) Anti-DNA topoisomerase I antibody □ (D) Anti-glomerular basement membrane antibody □ (E) Antihistone antibody □ (F) ANCA □ (G) Antiribonucleoprotein

Anti-double-stranded DNA antibody

A previously healthy, 21-year-old man sees his physician because he notices blood in his urine. He reports no dysuria, frequency, or hesitancy of urination. On physical examination, there are no abnormal findings. Laboratory findings show a serum urea nitrogen level of 39 mg/dL and creatinine level of 4.1 mg/dL. A renal biopsy specimen is obtained; the immunofluorescence pattern of staining with antibody against human IgG is shown in the figure. Which of the following serum laboratory studies is most likely to be positive in this patient? □ (A) Antistreptolysin O antibody □ (B) HIV antibody □ (C) Anti-glomerular basement membrane antibody □ (D) Hepatitis B surface antibody □ (E) C3 nephritic factor

Anti-glomerular basement membrane antibody

A 45-year-old man has experienced increasing malaise, nausea, and reduced urine output for the past 3 days. On physical examination, he is afebrile and normotensive. Laboratory findings show a serum creatinine level of 2.5 mg/dL. Urinalysis shows hematuria, but no pyuria or glucosuria. A renal biopsy is done; the immunofluorescence pattern with antifibrinogen is shown in the figure. Which of the following additional studies is most useful for classification and treatment of this disease? □ (A) ANA titer □ (B) Anti-glomerular basement membrane antibody test □ (C) HIV titer □ (D) Quantitative serum immunoglobulins □ (E) Rheumatoid factor □ (F) Urine immunoelectrophoresis

Anti-glomerular basement membrane antibody test

A 32-year-old man has developed a fever and skin rash over the past 3 days. Five days later, he has increasing malaise and visits his physician. On physical examination, the maculopapular erythematous rash on his trunk has nearly faded away. His temperature is 37.1°C, and blood pressure is 135/85 mm Hg. Laboratory studies show a serum creatinine level of 2.8 mg/dL and blood urea nitrogen level of 29 mg/dL. Urinalysis shows 2+ proteinuria; 1+ hematuria; and no glucose, ketones, or nitrite. The leukocyte esterase result is positive. Microscopic examination of urine shows RBCs and WBCs, some of which are eosinophils. What is the most likely cause of this patient's condition? □ (A) Urinary tract infection □ (B) Congestive heart failure □ (C) Antibiotic use □ (D) Streptococcal pharyngitis □ (E) Poorly cooked ground beef

Antibiotic use

A 9-year-old boy has developed prominent bruises on his extremities over the past week. On physical examination, he has ecchymoses and petechiae on his arms and legs. Laboratory studies show hemoglobin, 13.8 g/dL; hematocrit, 41.9%; MCV, 93 μm3; platelet count, 22,300/mm3; and WBC count, 7720/mm3. He had respiratory syncytial virus pneumonia 3 weeks ago. His condition improves with corticosteroid therapy. Which of the following abnormalities is most likely to cause his hemorrhagic diathesis? □ (A) Antiplatelet antibodies □ (B) Bone marrow aplasia □ (C) Glycoprotein IIb/IIIa dysfunction □ (D) Vitamin C deficiency □ (E) Von Willebrand factor metalloproteinase deficiency

Antiplatelet antibodies

A 29-year-old, HIV-positive woman has developed fever, cough, and dyspnea over the past week. On physical examination, her temperature is 37.9°C. There is dullness to percussion over lung fields posteriorly. A bronchoalveolar lavage is performed, and cysts of Pneumocystis carinii are present. She is given trimethoprim/sulfamethoxazole. One week later, her respiratory status has improved. Laboratory studies now show hemoglobin, 7.4 g/dL; hematocrit, 22.2%; MCV, 98 μm3; platelet count, 47,000/mm3; and WBC count, 1870/mm3 with 2% segmented neutrophils, 2% bands, 85% lymphocytes, 10% monocytes, and 1% eosinophils. One week later, she experiences increasing dyspnea, and a chest CT scan shows multiple 1- to 3-cm nodules with hemorrhagic borders in all lung fields. These nodules are most likely to be caused by infection with which of the following organisms? □ (A) Aspergillus fumigatus □ (B) Bartonella henselae □ (C) Mycobacterium avium complex □ (D) Escherichia coli □ (E) Herpes simplex virus □ (F) Pneumocystis carinii □ (G) Toxoplasma gondii

Aspergillus fumigatus

A 34-year-old man has experienced multiple nosebleeds along with bleeding gums for the past month. On examination, his temperature is 37.3°C. He has multiple cutaneous ecchymoses. Laboratory studies show hemoglobin, 8.5 g/dL; hematocrit, 25.7%; platelet count, 13,000/mm3; and WBC count, 52,100/mm3 with 5% segmented neutrophils, 5% bands, 2% myelocytes, 83% blasts, 3% lymphocytes, and 2% monocytes. Examination of his peripheral blood smear shows the blasts have delicate nuclear chromatin along with fine cytoplasmic azurophilic granules. These blasts are CD33+. Which of the following morphologic findings is most likely to be present on his peripheral blood smear? □ (A) Auer rods □ (B) Döhle bodies □ (C) Hairy projections □ (D) Heinz bodies □ (E) Sickle cells □ (F) Toxic granulations

Auer rods

A 45-year-old man has experienced a gradual weight loss and weakness, anorexia, and easy fatigability for 7 months. Physical examination shows marked splenomegaly. CBC shows hemoglobin, 12.9 g/dL; hematocrit, 38.1%; MCV, 92 μm3; platelet count, 410,000/mm3; and WBC count, 168,000/mm3. The peripheral blood smear is depicted in part A of the figure. Karyotypic analysis shows the Ph1 chromosome. The patient undergoes chemotherapy with imatinib mesylate (tyrosine kinase inhibitor), which reduces the spleen size and brings the total leukocyte count within normal range. He remains in remission for 3 years and then begins to experience fatigue and a 10-kg weight loss. Physical examination now shows petechial hemorrhages. CBC shows hemoglobin, 10.5 g/dL; hematocrit, 30%; platelet count, 60,000/μL; and WBC count, 40,000/μL. A peripheral blood smear is depicted in part B of the figure. Karyotypic analysis shows two Ph1 chromosomes and aneuploidy. Flow cytometric analysis of the peripheral blood shows CD19+, CD10+, sIg−, and CD3− cells. Which of the following complications of the initial disease did this patient develop after therapy? □ (A) Sézary syndrome □ (B) Myelodysplastic syndrome □ (C) Hairy cell leukemia □ (D) B lymphoblastic leukemia □ (E) Acute myeloblastic leukemia

B lymphoblastic leukemia

A 9-year-old, otherwise healthy girl has complained of pain in the right armpit for the past week. Examination by the physician shows tender lymphadenopathy of the right axillary region. There are four linear and nearly healed abrasions over a 3 × 2 cm area of the distal ventral aspect of the right forearm and a single, 0.5-cm, slightly raised erythematous nodule over one of the abrasions. No other abnormalities are noted. If a lymph node biopsy were performed, the microscopic appearance of the specimen would show a pattern of stellate, necrotizing granulomas. The lymphadenopathy regresses over the next 2 months. Infection with which of the following is most likely to have produced these findings? □ (A) Bartonella henselae □ (B) Cytomegalovirus □ (C) Epstein-Barr virus □ (D) Staphylococcus aureus □ (E) Yersinia pestis

Bartonella henselae

A 69-year-old woman complains of increasing back pain for 1 month. On physical examination, there is tenderness over the lower back, but no kyphosis or scoliosis. A radiograph of the spine shows a partial collapse of T11 and several 0.5- to 1.5-cm lytic lesions with a rounded "soap-bubble" appearance in the thoracic and lumbar vertebrae. A bone marrow biopsy is performed, and a smear of the aspirate is shown in the figure. Which of the following is the most likely laboratory finding in this patient? □ (A) Bence Jones proteins in the urine □ (B) t(9;22) in the karyotype of marrow □ (C) Elevated leukocyte alkaline phosphatase score □ (D) Decreased serum alkaline phosphatase level □ (E) Platelet count of 750,000/mm3 □ (F) WBC count of 394,000/mm3

Bence Jones proteins in the urine

A 66-year-old woman died of an acute myocardial infarction. At autopsy, both kidneys were decreased in size (about 120 g each) with a finely granular cortical surface. The representative appearance of the kidney under high magnification is shown in the figure. Which of the following clinical abnormalities most likely accompanied this lesion? □ (A) Oliguria □ (B) Benign hypertension □ (C) Malignant hypertension □ (D) Hematuria □ (E) Flank pain

Benign hypertension

A 65-year-old man diagnosed with follicular non-Hodgkin's lymphoma is treated with chemotherapy. He develops fever and cough. On examination, there are bilateral pulmonary rales. A chest radiograph shows diffuse interstitial infiltrates. A shell vial assay of sputum is positive for cytomegalovirus. He develops scleral icterus. Laboratory studies show hemoglobin, 10.3 g/dL; hematocrit, 41.3%; MCV, 101 μm3; WBC count, 7600/mm3; and platelet count, 205,000/mm3. His serum total bilirubin is 6 mg/dL, direct bilirubin is 0.8 mg/dL, and LDH is 1020 U/L. Coombs's test is positive. Which of the following is the most likely mechanism for his anemia? □ (A) Binding of IgM to red blood cells □ (B) Chemotherapeutic bone marrow toxicity □ (C) Cytomegalovirus hepatitis □ (D) Dietary folate deficiency □ (E) Metastases to colon

Binding of IgM to red blood cells

The most common vector borne disease in the United States is caused by which of the following pathogens? (A) Plasmodium vivax (B) Babesia microti (C) Borrelia burgdorferi (D) Borrelia hermsii (E) Rickettsia africae

Borrelia burgdorferi

A 48-year-old man has noticed a reddish area on the penis for the past 3 months. He has had no sexual intercourse for more than 1 month. On physical examination, there is a solitary 0.8-cm, plaquelike, erythematous area on the distal shaft of the penis. A routine microbiologic culture with a Gram-stained smear of the lesion shows normal skin flora. Microscopic examination of a biopsy specimen of the lesion shows dysplasia involving the full thickness of the epithelium. What is the most likely diagnosis? □ (A) Primary syphilis □ (B) Balanitis □ (C) Soft chancre □ (D) Bowen disease □ (E) Condyloma acuminatum

Bowen disease

A 46-year-old man with a history of poorly controlled diabetes mellitus comes to the physician because he has had painful, erosive, markedly pruritic lesions on the glans penis, scrotum, and inguinal regions of the skin for the past 2 months. Physical examination shows irregular, shallow, 1- to 4-cm erythematous ulcerations. Scrapings of the lesions are examined under the microscope. Which of the following microscopic findings in the scrapings is most likely to be reported? □ (A) Eggs and excrement of Sarcoptes scabiei □ (B) Budding cells with pseudohyphae □ (C) Atypical cells with hyperchromatic nuclei □ (D) Enlarged cells with intranuclear inclusions □ (E) Spirochetes under dark-field examination

Budding cells with pseudohyphae

A 12-year-old boy is taken to the physician because he has had increasing abdominal distention and pain for the past 3 days. Physical examination shows lower abdominal tenderness, and the abdomen is tympanitic with reduced bowel sounds. An abdominal CT scan shows a 7-cm mass involving the region of the ileocecal valve. Surgery is performed to remove the mass. Histologic examination of the mass shows sheets of intermediate-sized lymphoid cells, with nuclei having coarse chromatin, several nucleoli, and many mitoses. A bone marrow biopsy sample is negative for this cell population. Cytogenetic analysis of the cells from the mass shows a t(8;14) karyotype. Flow cytometric analysis reveals 40% of the cells are in S phase. The tumor shrinks dramatically in size after a course of chemotherapy. Which of the following is the most likely diagnosis? □ (A) Diffuse large B-cell lymphoma □ (B) Follicular lymphoma □ (C) Acute lymphoblastic leukemia □ (D) Plasmacytoma □ (E) Burkitt lymphoma

Burkitt lymphoma

A 41-year-old man has experienced several bouts of pneumonia over the past year. He now complains of vague abdominal pain and a dragging sensation. Physical examination shows marked splenomegaly. CBC shows hemoglobin, 8.2 g/dL; hematocrit, 24.6%; MCV, 90 μm3; platelet count, 63,000/mm3; and WBC count, 2400/mm3. The peripheral blood smear shows many small leukocytes with reniform nuclei and pale blue cytoplasm with threadlike extensions. A chest x-ray shows patchy infiltrates, and a culture of sputum grows Mycobacterium kansasii. Which of the following laboratory findings is most characteristic of this disease? □ (A) CD19, CD20, and CD11c expression in leukocytes □ (B) Presence of Auer rods in leukocytes □ (C) Presence of Ph1 chromosome □ (D) Presence of toxic granulations in neutrophils □ (E) Monoclonal IgM in serum

CD19, CD20, and CD11c expression in leukocytes

A 51-year-old man visits his physician because the skin of his face, neck, and trunk has become scaly red. He also complains of intense itching and a 3-kg weight loss over the past 2 months. On physical examination, his temperature is 37.6°C, and he has a generalized exfoliative erythroderma. Generalized nontender lymphadenopathy is present. Laboratory studies show hemoglobin, 12.9 g/dL; hematocrit, 42%; platelet count, 231,000/mm3; and WBC count, 7940/mm3 with 57% segmented neutrophils, 3% bands, 26% lymphocytes, 5% monocytes, and 9% eosinophils. A skin biopsy specimen shows the presence of lymphoid cells in the upper dermis and epidermis. These cells have cerebriform nuclei with marked infolding of nuclear membranes. Similar cells are seen on the peripheral blood smear. Which combination of the following phenotypic markers is most likely to be expressed on his abnormal lymphocytes? □ (A) CD3+, CD4+ □ (B) CD5+, CD56+ □ (C) CD10+, CD19+ □ (D) CD13+, CD33+ □ (E) CD19+, sIg+

CD3+, CD4+

A 24-year-old man is awakened at night because of severe lower abdominal pain that radiates to the groin. The pain is very intense and comes in waves. The next morning, he notices blood in his urine. He has no underlying illnesses and has been healthy all his life. On physical examination, he is afebrile and has a blood pressure of 110/70 mm Hg. Laboratory studies show serum Na+, 142 mmol/L; K+, 4 mmol/L; Cl−, 96 mmol/L; CO2, 25 mmol/L; glucose, 74 mg/dL; creatinine, 1.1 mg/dL; calcium, 9.1 mg/dL; and phosphorus, 2.9 mg/dL. Urinalysis shows a pH of 7; specific gravity of 1.020; and no protein, glucose, ketones, or nitrite. The patient is advised to drink more water. He likes iced tea and consumes large quantities over the course of a hot summer. He continues to have similar episodes. Which of the following substances is most likely to be increased in his urine? □ (A) Calcium oxalate □ (B) Cystine □ (C) Magnesium ammonium phosphate □ (D) Mucoprotein □ (E) Uric acid

Calcium oxalate

A sexually active, 26-year-old man has had pain on urination for the past 4 days. On physical examination, there are no lesions on the penis. He is afebrile. Urinalysis shows no blood, ketones, protein, or glucose. Microscopic examination of the urine shows few WBCs and no casts or crystals. What infectious agent is most likely to produce these findings? □ (A) Chlamydia trachomatis □ (B) Mycobacterium tuberculosis □ (C) Herpes simplex virus □ (D) Candida albicans □ (E) Treponema pallidum

Chlamydia trachomatis

A 23 year old Duke Medical student presents to the Emergency Room with two days of subjective fever and chills. Upon questioning, she reports returning from Tanzania ten days ago. She did not take antimalarial prophylaxis during her previous 2-week stay in Kenya. Physical examination is remarkable for fever 39.5°C and a tender, palpable spleen tip. Initial laboratory values are remarkable for hematocrit of 30%, mild thrombocytopenia (90,000/mm3), white blood cell count of 3500/μL with 25% immature band forms. Which of the following statements is false? (A) This returning traveler should be considered to have malaria until proven otherwise. (B) Malaria is the #1 life threatening infection among returning travelers. (C) Death from malaria in the US and other developed countries is often related to failure to consider the diagnosis of malaria in a timely fashion. (D) Chloroquine is likely to be effective therapy for this patient.

Chloroquine is likely to be effective therapy for this patient

A 42-year-old man has experienced increasing malaise for the past month. He is bothered by increasing swelling in the hands and legs. On physical examination, there is generalized edema. He is afebrile, and his blood pressure is 140/90 mm Hg. Urinalysis shows a pH of 6.5; specific gravity 1.017; 4+ proteinuria; and no blood, glucose, or ketones. Microscopic examination of the urine shows no casts or RBCs and 2 WBCs per high-power field. The 24-hour urine protein level is 4.2 g. A renal biopsy specimen is obtained, and immunofluorescence staining with antibody to the C3 component of complement produces the pattern shown in the figure. Which of the following underlying disease processes is most likely to be present? □ (A) Chronic hepatitis B □ (B) AIDS □ (C) Multiple myeloma □ (D) Recurrent urinary tract infection □ (E) Nephrolithiasis

Chronic Hepatitis B

A 55-year-old man has dysuria, increased frequency, and urgency of urination for the past 6 months. He has sometimes experienced mild lower back pain. On physical examination, he is afebrile. There is no costovertebral angle tenderness. The prostate gland feels normal in size; no nodules are palpable. Laboratory studies show that expressed prostatic secretions contain 30 leukocytes per high-power field. What is the most likely diagnosis? □ (A) Benign prostatic hyperplasia □ (B) Acute bacterial prostatitis □ (C) Syphilitic prostatitis □ (D) Chronic abacterial prostatitis □ (E) Metastatic prostatic adenocarcinoma

Chronic abacterial prostatitis

A 65-year-old man has experienced worsening fatigue for the past 5 months. On physical examination, he is afebrile and has a pulse of 91/min, respirations of 18/min, and blood pressure of 105/60 mm Hg. There is no organomegaly. A stool sample is positive for occult blood. Laboratory findings include hemoglobin of 5.9 g/dL, hematocrit of 17.3%, MCV of 96 μm3, platelet count of 250,000/mm3, and WBC count of 7800/mm3. The reticulocyte concentration is 3.9%. No fibrin split products are detected, and direct and indirect Coombs' test results are negative. A bone marrow biopsy specimen shows marked erythroid hyperplasia. Which of the following conditions best explains these findings? □ (A) Chronic blood loss □ (B) Iron deficiency anemia □ (C) Aplastic anemia □ (D) Metastatic prostatic adenocarcinoma □ (E) Autoimmune hemolytic anemia

Chronic blood loss

A 58-year-old woman sees her physician for a routine health maintenance examination. The only abnormality on physical examination is a blood pressure of 168/109 mm Hg. Urinalysis shows a pH of 7.0; specific gravity 1.020; 1+ proteinuria; and no blood, glucose, or ketones. An abdominal ultrasound scan shows bilaterally and symmetrically small kidneys with no masses. The ANA test result is negative. The serum urea nitrogen level is 51 mg/dL, and the creatinine level is 4.7 mg/dL. The hemoglobin A1c concentration is within the reference range. What is the most likely diagnosis? □ (A) Lupus nephritis □ (B) Autosomal-dominant polycystic kidney disease □ (C) Chronic glomerulonephritis □ (D) Nodular glomerulosclerosis □ (E) Amyloidosis

Chronic glomerulonephritis

A 70 year old man has experienced increasing fatigue for the past six months. On physical examination, he has nontender axillary and cervical lymphadenopathy, but there is mild splenomegaly. The hematologic workup yields hemoglobin 9.3 g/dL, hematocrit 28%, MCV 90 fL, platelet count 120,000/mm3, and WBC count 42,000/mm3. The peripheral blood smear shows a monotonous population of small, round, mature-looking lymphocytes. Flow cytometry shows these cells to be CD19+, CD5+, CD23+ and TdT-. Which of the following is the most likely diagnosis? (A) Chronic myelogenous leukemia (B) Chronic lymphocytic leukemia (C) Acute lymphocytic leukemia (D) Acute myelogenous leukemia (E) Non-Hodgkin's lymphoma

Chronic lymphocytic leukemia

The figure skater Sonja Henie, who won gold medals at the 1928, 1932, and 1936 Winter Olympic Games, became progressively fatigued in her late 50s. On physical examination, she had palpable nontender axillary and inguinal lymph nodes, and the spleen tip was palpable. Laboratory studies showed hemoglobin, 10.1 g/dL; hematocrit, 30.5%; MCV, 90 μm3; platelet count, 89,000/mm3; and WBC count, 31,300/mm3. From the peripheral blood picture shown in the figure, which of the following is the most likely diagnosis? □ (A) Infectious mononucleosis □ (B) Chronic lymphocytic leukemia □ (C) Iron deficiency anemia □ (D) Leukemoid reaction □ (E) Acute lymphoblastic leukemia

Chronic lymphocytic leukemia

A 63 year man complains of progressive fatigue. Physical examination reveals a markedly enlarged spleen. Laboratory studies reveal Hct 31%, WBC count 125,000/mm3, and platelet count 487,000/mm3. Cytogenetic analysis yields t (9;22) (q34;q11). A peripheral blood smear is shown below. Which of the following is the most likely diagnosis? (A) Acute lymphoblastic leukemia (B) Acute myelogenous leukemia M5 (C) Chronic myelogenous leukemia (D) Diffuse large cell lymphoma (E) Myelodysplasia

Chronic myelogenous leukemia

In an experiment, cell samples are collected from the bone marrow aspirates of patients who were diagnosed with lymphoproliferative disorders. Cytogenetic analyses are performed on these cells, and a subset of the cases is found to have the BCR-ABL fusion gene from the reciprocal translocation t(9;22)(q34;11). The presence of this gene results in increased tyrosine kinase activity. Patients with which of the following conditions are most likely to have this gene? □ (A) Follicular lymphoma □ (B) Chronic myelogenous leukemia □ (C) Hodgkin lymphoma, lymphocyte depletion type □ (D) Acute promyelocytic leukemia □ (E) Multiple myeloma

Chronic myelogenous leukemia

A 28-year-old woman has had dysuria, frequency, and urgency for the past 2 days. On physical examination, her temperature is 37.6°C. A urine culture grows greater than 100,000 colonies/mL of Escherichia coli. She is treated with antibiotic therapy. If the problem continues to recur, the patient is likely to be at greatest risk for development of which of the following renal diseases? □ (A) Diffuse glomerulosclerosis □ (B) Chronic glomerulonephritis □ (C) Amyloidosis □ (D) Membranous glomerulonephritis □ (E) Chronic pyelonephritis

Chronic pyelonephritis

A 60-year-old man has had increasing abdominal discomfort and fatigue for the past 9 months. He has noted easy bruising of his skin with minor trauma for the past month. On examination, he is afebrile, but his spleen is palpable and tender. Laboratory studies show hemoglobin, 7.7 g/dL; hematocrit, 23%; platelet count, 30,000/mm3; and WBC count, 2300/mm3 with 45% polymorphonuclear leukocytes, 50% lymphocytes, and 5% monocytes. His serum AST is 71 U/L, and ALT is 82 U/L. What is the most likely diagnosis? □ (A) Acute myelogenous leukemia □ (B) Cirrhosis □ (C) Infectious mononucleosis □ (D) Niemann-Pick disease □ (E) Metastatic adenocarcinoma □ (F) Systemic lupus erythematosus

Cirrhosis

A 63-year-old man has noticed a lump in her neck for 2 months. Examination reveals a group of three discrete nontender right posterior cervical lymph nodes, and a mass of enlarged right axillary lymph nodes. Chest and abdominal CT scans show mediastinal lymphadenopathy and hepatosplenomegaly. A cervical lymph node biopsy reveals abundant large CD15+ and CD30+ binucleate cells with prominent acidophilic nucleoli, scattered within a sparse lymphocytic infiltrate. What is molecular analysis of this lesion most likely to reveal? □ (A) Clonal EBV integration in the large cells □ (B) BCL6 gene rearrangements in the large cells □ (C) Deletions of 5q in all the cells □ (D) Helicobacter pylori infection in all the cells □ (E) JAK2 gene mutations in the lymphocytes

Clonal EBV integration in the large cells

A 32-year-old woman visits her physician because she has experienced fatigue, fever, night sweats, and painless lumps in the right side of her neck for the past 3 months. On physical examination, her temperature is 37.5°C. She has right cervical nontender lymphadenopathy. One of the lymph nodes is biopsied, and the histologic finding is shown at high power in the figure. Molecular analysis of large cells exemplified by the cell of the center is most likely to reveal which of the following genetic abnormalities? □ (A) Clonal rearrangement of T-cell receptor genes □ (B) Clonal rearrangement of immunoglobulin genes □ (C) Polyclonal rearrangement of T-cell receptor genes □ (D) Polyclonal rearrangement of immunoglobulin genes □ (E) Integration of the cytomegalovirus genome □ (F) Integration of the human herpesvirus-8 genome □ (G) Integration of the HTLV-1 genome

Clonal rearrangement of immunoglobulin genes

A 70-year-old man has experienced increasing fatigue for the past 6 months. On physical examination, he has nontender axillary and cervical lymphadenopathy, but there is no hepatosplenomegaly. The hematologic work-up shows hemoglobin, 9.5 g/dL; hematocrit, 28%; MCV, 90 μm3; platelet count, 120,000/mm3; and WBC count, 42,000/mm3. The peripheral blood smear shows a monotonous population of small, round, mature-looking lymphocytes. Flow cytometry shows these cells to be CD19+, CD5+, and TdT−. Which of the following is most likely to be seen with cytogenetic and molecular analysis of the cells in the patient's blood? □ (A) t(9;22) leading to BCR-ABL rearrangement □ (B) Clonal rearrangement of immunoglobulin genes □ (C) Clonal rearrangement of T-cell receptor genes □ (D) t(8;14) leading to C-MYC overexpression □ (E) t(14;18) leading to BCL2 overexpression

Clonal rearrangement of immunoglobulin genes

A 23-year-old, previously healthy man has experienced malaise and a low-grade fever and sore throat for 2 weeks. On physical examination, his temperature is 37.6°C, and he has pharyngeal erythema without exudation. There is tender cervical, axillary, and inguinal lymphadenopathy. Laboratory studies show hemoglobin, 12.2 g/dL; hematocrit, 36.6%; platelet count, 190,200/mm3; and WBC count, 8940/mm3. The peripheral blood smear is shown in the figure. Which of the following is the most likely risk factor for the disease that would be diagnosed in this patient? □ (A) An inherited disorder of globin chain synthesis □ (B) Transfusion of packed RBCs □ (C) Close personal contact (kissing) with his date □ (D) Sharing infected needles for intravenous drug use □ (E) Ingestion of raw oysters

Close personal contact (kissing) with his date

A 22-year-old woman has experienced malaise and a sore throat for 2 weeks. Her fingers turn white on exposure to cold. On physical examination, she has a temperature of 37.8°C, and the pharynx is erythematous. Laboratory findings include a positive Monospot test result. Direct and indirect Coombs test results are positive at 4°C, although not at 37°C. Which of the following substances on the surfaces of the RBCs most likely accounts for these findings? □ (A) IgE □ (B) Complement C3b □ (C) Histamine □ (D) IgG □ (E) Fibronectin

Complement C3b

A 7-year-old boy is recovering from impetigo. Physical examination shows a few honey-colored crusts on his face. The crusts are removed, and a culture of the lesions grows group A Streptococcus pyogenes. He is treated with antibiotics. One week later, he develops malaise with nausea and a slight fever and passes dark brown urine. Laboratory studies show a serum antistreptolysin O titer of 1:1024. Which of the following is the most likely outcome? □ (A) Development of rheumatic heart disease □ (B) Chronic renal failure □ (C) Lower urinary tract infection □ (D) Complete recovery without treatment □ (E) Progression to crescentic glomerulonephritis

Complete recovery without treatment

A 7-year-old boy is brought to the physician by his mother, who is concerned because he has become less active over the past 10 days. On physical examination, the boy has facial puffiness. Urinalysis shows no blood, glucose, or ketones, and microscopic examination shows no casts or crystals. The serum creatinine level is normal. A 24-hour urine collection yields 3.8 g of protein. He improves after corticosteroid therapy. He has two more episodes of proteinuria over the next few years, both of which respond to corticosteroid therapy. A renal biopsy is done. What is the most likely mechanism causing his disease? □ (A) Immune complex-mediated glomerular injury □ (B) Verocytotoxin-induced endothelial cell injury □ (C) Cytotoxic T cell-mediated tubular epithelial cell injury □ (D) Cytokine-mediated visceral epithelial cell injury □ (E) IgA-mediated mesangial cell injury

Cytokine-mediated visceral epithelial cell injury

A clinical study of patients who inherit mutations that reduce the level of spectrin in the RBC membrane cytoskeleton shows an increased prevalence of chronic anemia with splenomegaly. For many patients, it is observed that splenectomy reduces the severity of anemia. This beneficial effect of splenectomy is most likely related to which of the following processes? □ (A) Increase in synthesis of spectrin in RBCs □ (B) Increase in deformability of RBCs □ (C) Decrease in opsonization of RBCs □ (D) Decrease in trapping of RBCs in the spleen □ (E) Decrease in production of reactive oxygen species

Decrease in trapping of RBCs in the spleen

One week after a mild flulike illness, a 9-year-old boy has an episode of hematuria that subsides within 2 days. One month later, he tells his parents that his urine is red again. On physical examination, there are no significant findings. Urinalysis shows a pH of 7; specific gravity 1.015; 1+ proteinuria; 1+ hematuria; and no ketones, glucose, or urobilinogen. The serum urea nitrogen level is 36 mg/dL, and the creatinine level is 3.2 mg/dL. A renal biopsy specimen shows diffuse mesangial proliferation and electron-dense deposits in the mesangium. Which of the following mechanisms is most likely to produce these findings? □ (A) Deposition of immune complexes containing IgA □ (B) Formation of antibodies against type IV collagen □ (C) Virus-mediated injury to the glomeruli □ (D) Cytokine-mediated injury to the glomerular capillaries □ (E) Congenital defects in the structure of glomerular basement membranes

Deposition of immune complexes containing IgA

A 49-year-old man saw his physician because he had increased swelling in the extremities for 2 months. Physical examination showed generalized edema. A 24-hour urine collection yielded 4.1 g of protein and albumin and globulins. Extensive testing did not indicate the presence of a systemic disease, such as diabetes mellitus or systemic lupus erythematosus. He did not respond to a course of corticosteroid therapy. A renal biopsy was done, and microscopic examination showed diffuse thickening of the basement membrane. Immunofluorescence staining with antibody to the C3 component of complement was positive in a granular pattern in the glomerular capillary loops. Two years later, he experiences increasing malaise. Laboratory studies now show serum creatinine level of 4.5 mg/dL and urea nitrogen level of 44 mg/dL. Which of the following immunological mechanisms was most likely responsible for the glomerular changes observed in the biopsy specimen? □ (A) Antibodies that react with basement membrane collagen □ (B) Antibodies against streptococci that cross-react with the basement membrane □ (C) Release of cytokines by inflammatory cells □ (D) Cytotoxic T cells directed against renal antigens □ (E) Deposition of immune complexes on the basement membrane

Deposition of immune complexes on the basement membrane

For the past 6 months, a 35-year-old woman has experienced an excessively heavy menstrual flow each month. She also has noticed increasing numbers of pinpoint hemorrhages on her lower extremities in the past month. Physical examination shows no organomegaly or lymphadenopathy. CBC shows hemoglobin of 14.2 g/dL, hematocrit of 42.5%, MCV of 91 μm3, platelet count of 19,000/mm3, and WBC count of 6950/mm3. On admission to the hospital, she has melena and is given a transfusion of platelets, but her platelet count does not increase. An emergency splenectomy is performed, and her platelet count increases. Which of the following describes the most likely basis for her bleeding tendency? □ (A) Abnormalities in production of platelets by megakaryocytes □ (B) Suppression of pluripotent stem cells □ (C) Destruction of antibody-coated platelets by the spleen □ (D) Excessive loss of platelets in menstrual blood □ (E) Defective platelet-endothelial interactions

Destruction of antibody-coated platelets by the spleen

A 30 year old man complained of dyspnea, cough, and chest pain. His chest X-ray is shown below. A mediastinal biopsy was obtained, which is also shown below. Laboratory studies reveal Hgb 12 g/dl, WBC count 8250/mm3, and platelet count 255,000/mm3. LDH is elevated at 1117 U/L. Which of the following is the most likely diagnosis? (A) Chronic myelogenous leukemia (B) Acute myelogenous leukemia M5 (C) Follicular lymphoma (D) Diffuse large B cell lymphoma (E) Myelodysplasia

Diffuse large B cell lymphoma

A 53-year-old man comes to his physician because he felt a lump near his shoulder 1 week ago. On physical examination, there is an enlarged, nontender, supraclavicular lymph node and enlargement of the Waldeyer ring of oropharyngeal lymphoid tissue. There is no hepatosplenomegaly. CBC is normal except for findings of mild anemia. A lymph node biopsy specimen shows replacement by a monomorphous population of large lymphoid cells with enlarged nuclei and prominent nucleoli. Immunohistochemical staining and flow cytometry of the node indicates that most lymphoid cells are CD19+, CD10+, CD3−, CD15−, and terminal deoxynucleotidyl transferase negative (TdT−). Which of the following is the most likely diagnosis? □ (A) Chronic lymphadenitis □ (B) Diffuse large B-cell lymphoma □ (C) Hodgkin lymphoma □ (D) Lymphoblastic lymphoma □ (E) Small lymphocytic lymphoma

Diffuse large B-cell lymphoma

During the past 6 months, a 60-year-old man has noticed a malar skin rash that is made worse by sun exposure. He also has had arthralgias and myalgias. On physical examination, he is afebrile and has a pulse of 100/min, respirations of 20/min, and blood pressure of 100/60 mm Hg. There is erythema of skin over the bridge of the nose. No organomegaly is noted. Laboratory findings include positive serologic test results for ANA and double-stranded DNA, hemoglobin of 8.1 g/dL, hematocrit of 24.4%, platelet count of 87,000/mm3, and WBC count of 3950/mm3. The peripheral blood smear shows nucleated RBCs. A dipstick urinalysis is positive for blood, but there are no WBCs, RBCs, or casts seen on microscopic examination of the urine. Which of the following serum laboratory findings is most likely to be present? □ (A) Elevated D dimer □ (B) Negative Coombs antiglobulin test □ (C) Decreased iron □ (D) Elevated prostate-specific antigen □ (E) Diminished haptoglobin

Diminished haptoglobin

A 45-year-old Hispanic man has had increasing malaise with headaches and easy fatigability for the past 3 months. Physical examination reveals his blood pressure is 200/100 mm Hg. There are no palpable abdominal masses and no costovertebral tenderness. Laboratory studies show hemoglobin, 9.5 g/dL; hematocrit, 28.3%; MCV, 92 μm3; creatinine, 4.5 mg/dL; and urea nitrogen, 42 mg/dL. Urinalysis reveals 3+ hematuria and 3+ proteinuria, but no glucose or leukocytes. A renal biopsy is done; light microscopic examination of the biopsy specimen shows that approximately 50% of the glomeruli appear normal, but the rest show that a portion of the capillary tuft is sclerotic. Immunofluorescence staining shows IgM and C3 deposition in these sclerotic areas. Past history is significant for repeated episodes of passing dark brown urine, which failed to respond to corticosteroid therapy. Which of the following mechanisms is most likely responsible for his disease? □ (A) Deposition of immune complexes containing microbial antigens □ (B) Dysfunction of the podocyte slit diaphragm apparatus □ (C) Deposition of anti-glomerular basement membrane antibodies □ (D) Inherited defect in the basement membrane collagen □ (E) Deposition of C3 nephritic factor (C3NeF)

Dysfunction of the podocyte slit diaphragm apparatus

A 7-year-old boy has complained of a severe headache for the past week. On physical examination, there is tenderness on palpation of long bones, hepatosplenomegaly, and generalized lymphadenopathy. Petechial hemorrhages are present on the skin. Laboratory studies show hemoglobin, 8.8 g/dL; hematocrit, 26.5%; platelet count, 34,700/mm3; and WBC count, 14,800/mm3. A bone marrow biopsy specimen shows 100% cellularity, with almost complete replacement by a population of large cells with scant cytoplasm lacking granules, delicate nuclear chromatin, and rare nucleoli. He receives a course of chemotherapy and has a complete remission. Which of the following combinations of phenotypic and karyotypic markers is most likely to be present in this patient? □ (A) Early pre-B (CD19+, TdT+); hyperdiploidy □ (B) Early pre-B (CD19+, TdT+); t(9;22) □ (C) Pre-B (CD5+, TdT+); normal karyotype □ (D) Pre-B (CD5+, TdT+,); t(9;22) □ (E) T cell (CD3+, CD2+); normal karyotype □ (F) T cell (CD3+, CD2+); hyperdiploidy

Early pre-B (CD19+, TdT+); hyperdiploidy

The parents of a 6-year-old girl notice that she has become increasingly lethargic over the past 2 weeks. On examination by the physician, she has puffiness around the eyes. Her temperature is 36.9°C, and blood pressure is 100/60 mm Hg. Laboratory findings show a serum creatinine level of 0.7 mg/dL and urea nitrogen level of 12 mg/dL. Urinalysis shows a pH of 6.5; specific gravity 1.011; 4+ proteinuria; and no blood or glucose. The 24-hour urine protein level is 3.8 g. The child's condition improves after glucocorticoid therapy. Which of the following findings by electron microscopy is most likely to characterize this disease process? □ (A) Subepithelial electron-dense humps □ (B) Reduplication of glomerular basement membrane □ (C) Areas of thickened and thinned basement membrane □ (D) Increased mesangial matrix □ (E) Effacement of podocyte foot processes

Effacement of podocyte foot process

A 56-year-old man has had back pain and has passed dark-colored urine for the past month. On physical examination, there is right costovertebral angle tenderness. Urinalysis shows a pH of 6; specific gravity 1.015; 2+ hematuria; and no protein, glucose, or ketones. Microscopic examination of the urine shows numerous RBCs, few WBCs, and no casts or crystals. The figure shows the representative gross appearance of the renal lesion. Which of the following laboratory findings is most likely to be reported? □ (A) Elevated serum cortisol level □ (B) Elevated hematocrit □ (C) Ketonuria □ (D) Decreased creatinine clearance □ (E) Increased plasma renin activity

Elevated hematocrit

Over the past 9 months, a 30-year-old man has noticed increased heaviness with enlargement of the scrotum. On physical examination, there is an enlarged, firm left testis, but no other remarkable findings. An ultrasound scan shows a 5-cm solid mass within the body of the left testis. An orchiectomy of the left testis is performed. Microscopic examination of the mass shows areas of mature cartilage, keratinizing squamous epithelium, and colonic glandular epithelium. Laboratory findings include elevated levels of serum human chorionic gonadotropin (hCG) and α-fetoprotein (AFP). Despite the appearance of the cells in the tumor, the surgeon tells the patient that he probably has a malignant testicular tumor. The surgeon's conclusion is most likely based on which of the following factors? □ (A) Size of the tumor □ (B) Age of the patient □ (C) Presence of colonic glandular epithelium □ (D) Elevation of hCG and AFP levels □ (E) Location of the mass in the left testis

Elevation of hCG and AFP levels

A 45-year-old man comes to the physician for a routine health maintenance examination. On physical examination, there are no remarkable findings. Laboratory findings include serum creatinine, 1.1 mg/dL; urea nitrogen, 17 mg/dL; glucose, 76 mg/dL; alkaline phosphatase, 89 U/L; and prostate-specific antigen (PSA), 8 ng/mL. Prostate biopsies are performed and the high-power microscopic appearance of a biopsy specimen is shown in the figure. Which of the following is the most likely risk factor for his disease? □ (A) Epigenetic hypermethylation of GSTP1 gene □ (B) Recurrent bacterial urinary tract infections □ (C) Exposure to naphthylamine compounds □ (D) Tobacco use □ (E) Overproduction of dihydrotestosterone

Epigenetic hypermethylation of GSTP1 gene

A 37-year-old man known to have been infected with HIV for the past 10 years is admitted to the hospital with abdominal pain of 3 days' duration. Physical examination shows abdominal distention and absent bowel sounds. An abdominal CT scan shows a mass lesion involving the ileum. He undergoes surgery to remove an area of bowel obstruction in the ileum. Gross examination of the specimen shows a firm, white mass, 10 cm long and 3 cm at its greatest depth. The mass has infiltrated through the wall of the ileum. Histologic studies show a mitotically active population of CD19+ lymphoid cells with prominent nuclei and nucleoli. Molecular analysis is most likely to show which of the following viral genomes in the lymphoid cells? □ (A) Epstein-Barr virus □ (B) HIV □ (C) Human herpesvirus type 8 □ (D) Human T-cell leukemia/lymphoma virus type 1 □ (E) Cytomegalovirus

Epstein-Barr virus

A 9-year-old boy living in Uganda has had increasing pain and swelling on the right side of his face over the past 8 months. On physical examination, there is a large, nontender mass involving the mandible, which deforms the right side of his face. There is no lymphadenopathy and no splenomegaly, and he is afebrile. A biopsy of the mass is performed. Microscopically, the specimen is composed of intermediate-sized lymphocytes with a high mitotic rate. A chromosome analysis shows a 46,XY,t(8;14) karyotype in these cells. The hemoglobin concentration is 13.2 g/dL, platelet count is 272,000/mm3, and WBC count is 5820/mm3. Infection with which of the following is most likely to be causally related to the development of these findings? □ (A) Cytomegalovirus □ (B) Epstein-Barr virus □ (C) Hepatitis B virus □ (D) HIV □ (E) Human papillomavirus □ (F) Respiratory syncytial virus

Epstein-Barr virus

A 50-year-old woman has had fever and flank pain for the past 2 days. On physical examination, her temperature is 38.2°C, pulse is 81/min, respirations are 16/min, and blood pressure is 130/80 mm Hg. Urinalysis shows no protein, glucose, or ketones. The leukocyte esterase test is positive. Microscopic examination of the urine shows numerous polymorphonuclear leukocytes and occasional WBC casts. Which of the following organisms is most likely to be found in the urine culture? □ (A) Mycobacterium tuberculosis □ (B) Mycoplasma hominis □ (C) Escherichia coli □ (D) Group A streptococcus □ (E) Cryptococcus neoformans

Escherichia coli

A 50-year-old woman with diabetic nephropathy receives a renal allograft. An episode of acute cellular rejection requires an increase in immunosuppressive therapy. She develops dysuria. On examination, she has suprapubic pain on palpation. A urinalysis shows hematuria. Cystoscopy is performed, and 3- to 4-cm soft, yellow, slightly raised mucosal plaques are seen. Biopsy specimens of these lesions are taken and microscopically show mucosal infiltration by foamy macrophages with abundant PAS-positive cytoplasmic granules and small, laminated mineralized concretions. Which of the following organisms is most likely to be found in her urine? □ (A) Adenovirus □ (B) Candida albicans □ (C) Chlamydia trachomatis □ (D) Escherichia coli □ (E) Schistosoma haematobium

Escherichia coli

Several days after eating a hamburger, chili, and ice cream at a home barbecue, a 5-year-old girl develops cramping abdominal pain and diarrhea. The next day, she has decreased urine output. On physical examination, there are petechial hemorrhages on the skin. Her temperature is 37°C, pulse is 90/min, respirations are 18/min, and blood pressure is 90/50 mm Hg. A stool sample is positive for occult blood. Laboratory findings show hemoglobin, 10.8 g/dL; hematocrit, 32.4%; platelet count, 64,300/mm3; and WBC count, 6480/mm3. The peripheral blood smear shows schistocytes, and the serum D-dimer level is elevated. Which of the following is the most likely causative organism? □ (A) Candida albicans □ (B) Proteus mirabilis □ (C) Clostridium difficile □ (D) Escherichia coli □ (E) Staphylococcus aureus

Escherichia coli

A 63-year-old woman experiences a burning sensation in her hands and feet. Two months ago, she had an episode of swelling with tenderness in the right leg, followed by dyspnea and right-sided chest pain. On physical examination, the spleen and liver now appear to be enlarged. CBC shows hemoglobin, 13.3 g/dL; hematocrit, 40.1%; MCV, 91 μm3; platelet count, 657,000/mm3; and WBC count, 17,400/mm3. The peripheral blood smear shows abnormally large platelets. Which of the following is the most likely diagnosis? □ (A) Essential thrombocythemia □ (B) Chronic myelogenous leukemia □ (C) Myelofibrosis with myeloid metaplasia □ (D) Acute myelogenous leukemia □ (E) Polycythemia vera

Essential thrombocythemia

A 29-year-old man complains of a vague feeling of heaviness in the scrotum, but he has had no increase in pain for the past 5 months. He is otherwise healthy. Physical examination shows that the right testis is slightly larger than the left testis. An ultrasound scan shows the presence of a solid, circumscribed, 1.5-cm mass in the body of the right testis. The representative gross appearance of the mass is shown in the figure. A biopsy is done, and microscopic examination of the mass shows uniform nests of cells with distinct cell borders, glycogen-rich cytoplasm, and round nuclei with prominent nucleoli. There are aggregates of lymphocytes between these nests of cells. Which of the following features is most characteristic of this lesion? □ (A) Excellent response to radiation therapy □ (B) Likelihood of extensive metastases early in the course of disease □ (C) Elevation of human chorionic gonadotropin levels in the serum □ (D) Elevation of α-fetoprotein levels in the serum □ (E) Elevation of serum testosterone levels □ (F) Association with 46,X(fra)Y karyotype □ (G) Association with 46,XXY karyotype

Excellent response to radiation therapy

A 41-year-old woman sees her physician because of a 2-week history of multiple ecchymoses on her extremities after only minor trauma. She also reports feeling extremely weak. Over the previous 24 hours, she has developed a severe cough productive of yellowish sputum. On physical examination, her temperature is 38.4°C, and she has diffuse crackles on all lung fields. Laboratory studies show hemoglobin, 7.2 g/dL; hematocrit, 21.4%; MCV, 88 μm3; platelet count, 35,000/mm3; and WBC count, 1400/mm3 with 20% segmented neutrophils, 1% bands, 66% lymphocytes, and 13% monocytes. The reticulocyte count is 0.1%. Given these laboratory findings, which of the following historical findings would be most useful in determining the cause of her condition? □ (A) Exposure to drugs □ (B) Dietary history □ (C) Recent bacterial infection □ (D) Menstrual history □ (E) Family history of anemias

Exposure to drugs

A 65-year-old man sees his physician because he has experienced fatigue, a 5-kg weight loss, night sweats, and abdominal discomfort for the past year. On physical examination, he has marked splenomegaly; there is no lymphadenopathy. Laboratory studies show hemoglobin, 10.1 g/dL; hematocrit, 30.5%; MCV, 89 μm3; platelet count, 94,000/mm3; and WBC count, 14,750/mm3 with 55% segmented neutrophils, 9% bands, 20% lymphocytes, 8% monocytes, 4% metamyelocytes, 3% myelocytes, 1% eosinophils, and 2 nucleated RBCs per 100 WBCs. The peripheral blood smear also shows teardrop cells. The serum uric acid level is 12 mg/dL. A bone marrow biopsy specimen shows extensive marrow fibrosis and clusters of atypical megakaryocytes. Which of the following is most likely to account for the enlargement in this patient's spleen? □ (A) Hodgkin lymphoma □ (B) Extramedullary hematopoiesis □ (C) Portal hypertension □ (D) Granulomas with Histoplasma capsulatum □ (E) Metastatic adenocarcinoma

Extramedullary hematopoiesis

A clinical study of patients with hemoglobinopathies reveals that some of these patients developed episodes of acute chest, back, and abdominal pain; recurrent infections with Streptococcus pneumoniae; and chronic anemia as children and adults. They experienced none of these problems as infants, however. The presence of which of the following types of hemoglobin during infancy most likely provided protection from complications of their hemoglobinopathy? □ (A) A1 □ (B) A2 □ (C) C □ (D) E □ (E) F

F

An otherwise healthy, 72-year-old man has had increasing difficulty with urination for the past 10 years. He now has to get up several times each night because of a feeling of urgency, but each time the urine volume is not great. He has difficulty starting and stopping urination. On physical examination, the prostate is enlarged to twice its normal size, but is not tender to palpation. One year ago, his serum prostate-specific antigen (PSA) level was 6 ng/mL, and it is still at that level when retested. Which of the following drugs is most likely to be effective in treatment of this man? □ (A) Estrogen (hormone) □ (B) Finasteride (5-α-reductase inhibitor) □ (C) Mitoxantrone (chemotherapy agent) □ (D) Nitrofurantoin (antibiotic) □ (E) Prednisone (corticosteroid)

Finasteride (5-α-reductase inhibitor)

A 12-year-old girl has experienced increasing malaise for the past 2 weeks. On physical examination, she has periorbital edema. The child is afebrile. Laboratory findings show proteinuria on dipstick urinalysis, but no hematuria or glucosuria. Microscopic examination of the urine shows numerous oval fat bodies. The serum creatinine level is 2.3 mg/dL. She receives a course of corticosteroid therapy, but does not improve. A renal biopsy is done; the biopsy specimen shows that approximately 50% of the glomeruli in the specimen are affected by the lesion shown in the figure. What is the most likely diagnosis? □ (A) Focal segmental glomerulosclerosis □ (B) Lipoid nephrosis □ (C) Membranoproliferative glomerulonephritis type I □ (D) Membranoproliferative glomerulonephritis type II □ (E) Nodular glomerulosclerosis □ (F) Postinfectious glomerulonephritis □ (G) Rapidly progressive glomerulonephritis

Focal segmental glomerulosclerosis

The serum folate level must be checked before initiating therapy for a macrocytic anemia because (A) Excess folate is toxic (B) The folate level determines whether the macrocytic anemia is treated by dietary changes or a dietary supplement (C) Folate supplements mask the symptoms of pernicious anemia (D) The cause of the macrocytic anemia might be iron deficiency (E) A normal serum folate level means that the patient does not really have a macrocytic anemia

Folate supplements mask the symptoms of pernicious anemia

A 62 year old man presents to the Emergency Department in an obvious state of inebriation. He is well known there, because this scenario has been repeated many times over the years. On physical examination, he is afebrile. The spleen tip is palpable and the liver edge is firm. Laboratory studies yield hemoglobin 8.2 g/dL, hematocrit 25.1%, MCV 107 µm3, platelet count 135,000/mm3, and WBC count 3920/mm3. The peripheral blood smear reveals prominent anisocytosis and macrocytosis. Polychromatophilic RBCs are difficult to find. A few of the neutrophils have 6-7 nuclear lobes. Which of the following is the most likely explanation of these findings? (A) Mechanical fragmentation of RBCs (B) Autoimmune hemolytic anemia (C) Folic acid deficiency (D) Sickle cell disease (E) Reduced deformability of the RBC membrane

Folic acid deficiency

A 64-year-old man has inguinal, axillary, and cervical lymphadenopathy. The nodes are firm and nontender. A biopsy specimen of a cervical node shows a histologic pattern of nodular aggregates of small, cleaved lymphoid cells and larger cells with open nuclear chromatin, several nucleoli, and moderate amounts of cytoplasm. A bone marrow biopsy specimen shows lymphoid aggregates of similar cells with surface immunoglobulin that are CD10+, but CD5−. Karyotyping of these lymphoid cells indicates the presence of t(14;18). What is the most likely diagnosis? □ (A) Hodgkin lymphoma, nodular sclerosis type □ (B) Acute lymphadenitis □ (C) Follicular lymphoma □ (D) Mantle cell lymphoma □ (E) Toxoplasmosis

Follicular lymphoma

A 69-year-old, previously healthy woman has been feeling increasingly tired and weak for 4 months. On physical examination, she is afebrile. There is no hepatosplenomegaly or lymphadenopathy. Laboratory studies show hemoglobin of 9.3 g/dL, platelet count of 250,600/mm3, and WBC count of 6820/mm3. The appearance of the peripheral blood smear is shown in the figure. Which of the following conditions should be suspected as the most likely cause of these findings? □ (A) Pernicious anemia □ (B) Gastrointestinal blood loss □ (C) Aplastic anemia □ (D) β-Thalassemia major □ (E) Warm autoimmune hemolytic anemia

Gastrointestinal blood loss

Three days after taking an anti-inflammatory medication that includes phenacetin, a 23-year-old African-American man passes dark reddish brown urine. He is surprised by this because he has been healthy all his life and has had no major illnesses. On physical examination, he is afebrile, and there are no remarkable findings. CBC shows a mild normocytic anemia, but the peripheral blood smear shows precipitates of denatured globin (Heinz bodies) with supravital staining and scattered "bite cells" in the population of RBCs. Which of the following is the most likely diagnosis? □ (A) α-Thalassemia □ (B) Sickle cell trait □ (C) Glucose-6-phosphate dehydrogenase deficiency □ (D) Autoimmune hemolytic anemia □ (E) β-Thalassemia minor □ (F) RBC membrane abnormality

Glucose-6-phosphate dehydrogenase deficiency

A previously healthy 21 year old man sees his physician because he notices blood in his urine and he coughed up blood. He reports no dysuria, frequency, or hesitancy of urination. On physical examination, there are no abnormal findings. Laboratory findings yield a serum urea nitrogen of 39 mg/dL and creatinine of 4.1 mg/dL. A renal biopsy specimen is obtained, and the immunofluorescence pattern of staining with antibody against human IgG is shown below. No specific abnormality is seen on electron microscopy. Which of the following is the most likely diagnosis? (A) Membranous glomerulonephritis (B) Alport's disease (C) Goodpasture's syndrome (D) Focal segmental glomerulosclerosis (E) Diabetic nephropathy

Goodpasture's syndrome

A 58-year-old man from Nagasaki, Japan, has noted an increasing number of skin lesions for the past 8 months. On examination, there are scaling red-brown patches on all skin surfaces. He also has generalized lymphadenopathy and hepatosplenomegaly. Laboratory studies show hemoglobin, 9.7 g/dL; hematocrit, 31%; MCV, 89 μm3; platelet count, 177,000/mm3; and WBC count, 18,940/mm3 with differential count of 35 segmented neutrophils, 2 band neutrophils, 58 lymphocytes, and 5 monocytes. His serum calcium is 11.5 mg/dL. Examination of his peripheral blood smear shows multilobated "cloverleaf" cells. Despite aggressive chemotherapy, his condition worsens with development of paresthesias along with erythematous plaques and red-brown nodules on his skin. Which of the following infectious agents most likely caused his illness? □ (A) Cytomegalovirus □ (B) Epstein-Barr virus (EBV) □ (C) HIV □ (D) HTLV-1 □ (E) Rochalimaea henselae □ (F) Yersinia pestis

HTLV-1

A patient with cutaneous leishmaniasis acquired in Peru three months ago requests treatment for his lesion even though it seems to be healing well. You agree to treat him because (A) His father is a rich diplomat (B) His primary lesion will never heal without therapy (C) He is at risk of mucocutaneous disease (D) On second thought, you advise against treatment

He is at risk of mucocutaneous disease

A 5-year-old boy has had a history of easy bruising and blood in his urine since infancy. Physical examination shows no organomegaly. He has several ecchymoses of the skin on the lower extremities. Laboratory studies show hemoglobin, 13.1 g/dL; hematocrit, 39.3%; platelet count, 287,600/mm3; WBC count, 6830/mm3; prothrombin time, 13 seconds; partial thromboplastin time, 54 seconds; and less than 1% factor VIII activity measured in plasma. If he does not receive transfusions of recombinant factor VIII concentrate, which of the following manifestations of this illness is most likely to ensue? □ (A) Splenomegaly □ (B) Conjunctival petechiae □ (C) Hemolysis □ (D) Hemochromatosis □ (E) Hemarthroses

Hemarthroses

Soon after crossing the finish line in a 10-km race, a 31-year-old man collapses. On physical examination, his temperature is 40.1°C, pulse is 101/min, respirations are 22/min, and blood pressure is 85/50 mm Hg. He is not perspiring, and his skin shows tenting. Laboratory studies show Na+, 155 mmol/L; K+, 4.6 mmol/L; Cl−, 106 mmol/L; CO2, 27 mmol/L; glucose, 68 mg/dL; creatinine, 1.8 mg/dL; hemoglobin, 20.1 g/dL; hematocrit, 60.3%; platelet count, 230,400/mm3; and WBC count, 6830/mm3. What is the most likely diagnosis? □ (A) Erythroleukemia □ (B) Chronic obstructive pulmonary disease □ (C) Diabetes insipidus □ (D) Hemoconcentration □ (E) Increased erythropoietin levels □ (F) Polycythemia vera

Hemoconcentration

A 58-year-old, relatively healthy man sees his physician for a routine health maintenance examination. Physical examination shows mild hypertension. Laboratory findings show a serum creatinine level of 2.2 mg/dL and urea nitrogen level of 25 mg/dL. Microalbuminuria is present, with excretion of 250 mg/day of albumin. Two years later, he returns for a follow-up visit. He is now hypertensive and has a serum creatinine level of 3.8 mg/dL, urea nitrogen level of 38 mg/dL, and 24-hour urine protein level of 2.8 g. A renal biopsy is done; the light microscopic appearance of a PAS-stained specimen is shown in the figure. Which of the following laboratory findings is most likely to be abnormal in this patient? □ (A) Anti-glomerular basement membrane antibody □ (B) ANA □ (C) ANCA □ (D) Antistreptolysin O □ (E) C3 nephritic factor □ (F) Hemoglobin A1c □ (G) Hepatitis B surface antigen

Hemoglobin A1c

A 32-year-old woman from Saigon, Vietnam, gives birth at 34 weeks' gestation to a markedly hydropic stillborn male infant. Autopsy findings include hepatosplenomegaly and cardiomegaly, serous effusions in all body cavities, and generalized hydrops. No congenital anomalies are noted. There is marked extramedullary hematopoiesis in visceral organs. Which of the following findings is most likely to be present on hemoglobin electrophoresis of the fetal RBCs? □ (A) Hemoglobin A1 □ (B) Hemoglobin A2 □ (C) Hemoglobin Bart's □ (D) Hemoglobin C □ (E) Hemoglobin E □ (F) Hemoglobin F □ (G) Hemoglobin H □ (H) Hemoglobin S

Hemoglobin Bart's

An infant is born at 34 weeks' gestation to a 28-year-old woman, G3, P2. At birth, the infant is observed to be markedly hydropic and icteric. A cord blood sample is taken, and direct Coombs' test result is positive for the infant's RBCs. Which of the following is the most likely diagnosis? □ (A) Mechanical fragmentation of RBCs □ (B) Nuclear maturation defects resulting from impaired DNA synthesis □ (C) Impaired globin synthesis □ (D) Hemolysis of antibody-coated cells □ (E) Stem cell defect □ (F) Oxidative injury to hemoglobin □ (G) Reduced deformability of the RBC membrane

Hemolysis of antibody-coated cells

After eating a cheeseburger, French fries, and ice cream for dinner one night, a 6-year-old girl develops nausea, mild abdominal cramping, and a slight fever. Three days later, her parents notice that she is passing dark stools and dark urine and appears fatigued and weak. On physical examination, she has a temperature of 37.9°C, pulse of 88/min, respirations of 18/min, and blood pressure of 140/90 mm Hg. Scattered petechiae are present on the extremities. Laboratory findings show a serum creatinine level of 2.2 mg/dL and urea nitrogen level of 20 mg/dL. Urinalysis shows a pH of 6; specific gravity 1.016; 2+ hematuria; and no protein or glucose. A renal biopsy specimen shows small thrombi within glomerular capillary loops. Which of the following diseases is most likely to produce these findings? □ (A) Postinfectious glomerulonephritis □ (B) Wegener granulomatosis □ (C) Hereditary nephritis □ (D) Hemolytic-uremic syndrome □ (E) IgA nephropathy

Hemolytic-uremic syndrome

A 16-year-old girl has a history of easy bruising and hemorrhages. Since menarche at the age of 14 years, she has had menometrorrhagia. On physical examination, she displays joint deformity and has decreased mobility of the ankles, knees, and wrists. Laboratory studies show hemoglobin, 11.8 g/dL; hematocrit, 35.1%; platelet count, 267,000/mm3; WBC count, 5960/mm3; prothrombin time, 13 seconds; and partial thromboplastin time, 60 seconds. A 1 : 1 dilution of the patient's plasma with normal pooled plasma corrects the partial thromboplastin time. Which of the following is the most likely diagnosis? □ (A) Antiphospholipid syndrome □ (B) Disseminated intravascular coagulation □ (C) Hemophilia A □ (D) Idiopathic thrombocytopenic purpura □ (E) Thrombotic thrombocytopenic purpura □ (F) Vitamin K deficiency □ (G) Von Willebrand disease

Hemophilia A

A 13-year-old child has a history of easy bruising. At age 10 years, he experienced hemorrhaging around the pharynx that produced acute airway obstruction. On physical examination, he has marked reduction in joint mobility of the ankles, knees, and elbows. Family history indicates that other male relatives have similar bleeding problems. Laboratory studies show hemoglobin, 13.1 g/dL; hematocrit, 39.2%; platelet count, 228,000/mm3; WBC count, 5950/mm3; prothrombin time, 13 seconds; and partial thromboplastin time, 52 seconds. A 1 : 1 dilution of the patient's plasma with normal pooled plasma does not correct the partial thromboplastin time. Which of the following is the most likely diagnosis? □ (A) Antiphospholipid syndrome □ (B) Factor V mutation □ (C) Disseminated intravascular coagulation □ (D) Hemophilia A with a factor VIII inhibitor □ (E) Idiopathic thrombocytopenic purpura □ (F) Von Willebrand disease □ (G) Vitamin K deficiency

Hemophilia A with a factor VIII inhibitor

A 21-year-old woman known to have a protein C deficiency develops recurrent pulmonary thromboembolism and is placed on anticoagulant therapy. Two weeks after initiation of this therapy, she has a sudden change in mental status and experiences difficulty speaking and swallowing. A cerebral angiogram shows a distal left middle cerebral artery occlusion. Laboratory studies show hemoglobin of 13 g/dL, platelet count of 65,400/mm3, WBC count of 5924/mm3, prothrombin time of 12 seconds, and partial thromboplastin time of 51 seconds. The anticoagulant therapy is discontinued. Which of the following pharmacologic agents used as an anticoagulant in this patient is most likely to have caused these findings? □ (A) Acetylsalicylic acid (aspirin) □ (B) Warfarin □ (C) Heparin □ (D) Tissue plasminogen activator □ (E) Urokinase

Heparin

A clinical study is performed using patients diagnosed with peptic ulcer disease, chronic blood loss, and hypochromic microcytic anemia. Their serum ferritin levels average 5 to 7 μg/mL. The rate of duodenal iron absorption in this study group is found to be much higher than in a normal control group. After treatment with omeprazole and clarithromycin, study group patients have hematocrits of 40% to 42%, MCV of 82 to 85 μm3, and serum ferritin of less than 12 μg/mL. Measured rates of iron absorption in the study group after therapy are now decreased to the range of the normal controls. Which of the following substances derived from liver is most likely to have been increased in the study group patients before therapy, returning to normal after therapy? □ (A) Transferrin □ (B) Hemosiderin □ (C) Hepcidin □ (D) Divalent metal transporter-1 (DMT-1) □ (E) HLA-like transmembrane protein

Hepcidin

A 35-year-old, previously healthy man is found dead in his home. At autopsy, the medical examiner notices bilaterally enlarged kidneys that contain multiple, irregularly arranged cysts of different shapes and sizes. There is a 0.5-cm nonruptured intracerebral berry aneurysm of the anterior communicating artery. There are scattered 1- to 2-cm, fluid-filled liver cysts involving 10% of the parenchymal volume. Postmortem laboratory testing of the urine and blood shows markedly elevated levels of cocaine and its metabolite, benzoylecgonine. Which of the following is the most appropriate conclusion to be drawn from these findings? □ (A) He had lesions related to chronic use of cocaine □ (B) He had autosomal-recessive polycystic kidney disease, but survived to adulthood □ (C) His surviving family (children, siblings, and parents) should be evaluated for a similar condition □ (D) The immediate cause of death is berry aneurysm □ (E) The underlying cause of death is autosomal-dominant polycystic kidney disease

His surviving family (children, siblings, and parents) should be evaluated for a similar condition

An 18-month-old girl has developed seborrheic skin eruptions over the past 3 months. She has had recurrent upper respiratory and middle ear infections with Streptococcus pneumoniae for the past year. Physical examination indicates that she also has hepatosplenomegaly and generalized lymphadenopathy. Her hearing is reduced in the right ear. A skull radiograph shows an expansile, 2-cm lytic lesion involving the right temporal bone. Laboratory studies show no anemia, thrombocytopenia, or leukopenia. The mass is curetted. Which of the following is most likely to be seen on microscopic examination of this mass? □ (A) Histiocytes with Birbeck granules □ (B) Lymphoblasts □ (C) Plasma cells with Russell bodies □ (D) Reed-Sternberg cells □ (E) Ringed sideroblasts □ (F) Sézary cells

Histiocytes with Birbeck granules

A 60-year-old man presents with a feeling of fullness in his abdomen and a 5-kg weight loss over the past 6 months. Physical examination is normal. Laboratory studies show hemoglobin of 8.2 g/dL, hematocrit of 24%, and MCV of 70 μm3. Urinalysis shows 3+ hematuria, but no protein, glucose, or leukocytes. Abdominal CT scan shows an 11-cm mass in the upper pole of the right kidney. A right nephrectomy is performed, and on gross examination the mass invades the renal vein. Microscopic examination of the mass shows cells with abundant clear cytoplasm. Which of the following molecular abnormalities is most likely to be found in tumor cell DNA? □ (A) Homozygous loss of the von Hippel-Lindau (VHL) gene □ (B) Mutational activation of the MET proto-oncogene □ (C) Trisomy of chromosome 7 associated genes □ (D) Integration of human papillomavirus-16 (HPV-16) □ (E) Microsatellite instability

Homozygous loss of the von Hippel-Lindau (VHL) gene

A 25-year-old man has occasionally felt pain in the scrotum for the past 3 months. On physical examination, the right testis is more tender than the left, but does not appear to be appreciably enlarged. An ultrasound scan shows a 1.5-cm mass. A right orchiectomy is performed, and gross examination shows the mass to be hemorrhagic and soft. A retroperitoneal lymph node dissection is done. In sections of the lymph nodes, a neoplasm is seen with grossly extensive necrosis and hemorrhage. Microscopic examination shows that areas of viable tumor are composed of cuboidal cells intermingled with large eosinophilic syncytial cells containing multiple dark, pleomorphic nuclei. Immunohistochemical staining of the tumor is most likely to show which of the following antigenic components in the syncytial cells? □ (A) Human chorionic gonadotropin □ (B) α-Fetoprotein □ (C) Vimentin □ (D) CD20 □ (E) Testosterone □ (F) Carcinoembryonic antigen □ (G) CA-125

Human chorionic gonadotropin

A 23-year-old, sexually active man has been treated for Neisseria gonorrhoeae infection several times during the past 5 years. He now comes to the physician because of the increasing number and size of warty lesions slowly enlarging on his external genitalia during the past year. On physical examination, there are multiple 1- to 3-mm sessile, nonulcerated, papillary excrescences over the inner surface of the penile prepuce. These lesions are excised, but 2 years later, similar lesions appear. Which of the following conditions most likely predisposed him to development of these recurrent lesions? □ (A) Candida albicans infection □ (B) Circumcision □ (C) Human papillomavirus infection □ (D) Neisseria gonorrhoeae infection □ (E) Paraphimosis □ (F) Phimosis

Human papillomavirus infection

A 65-year-old woman has recently experienced several transient ischemic attacks. On physical examination, the only abnormal finding is a blood pressure of 150/95 mm Hg. Urinalysis shows 1+ proteinuria, and no glucose, blood, or ketones. Microscopic examination of the urine shows no RBCs or WBCs and few oxalate crystals. On abdominal ultrasound, the kidneys are slightly decreased in size. Which of the following renal lesions is most likely to be present in this patient? □ (A) Crescentic glomerulonephritis □ (B) Hyaline arteriolosclerosis □ (C) Mesangial cell proliferation □ (D) Arteriolar fibrinoid necrosis □ (E) Acute tubular necrosis □ (F) Interstitial nephritis

Hyaline arteriolosclerosis

A 59-year-old man notices gradual enlargement of the scrotum over the course of 1 year. The growth is not painful, but produces a sensation of heaviness. He has no problems with sexual function. Physical examination shows no lesions of the overlying scrotal skin and no obvious masses, but the scrotum is enlarged, boggy, and soft bilaterally. The transillumination test result is positive. What is the most likely diagnosis? □ (A) Varicocele □ (B) Elephantiasis □ (C) Orchitis □ (D) Seminoma □ (E) Hydrocele

Hydrocele

Three years ago, a 47-year-old woman had a mastectomy of the right breast to remove an infiltrating ductal carcinoma. She now has bone pain, and a radionuclide scan shows multiple areas of increased uptake in the vertebrae, ribs, pelvis, and right femur. Urinalysis shows a specific gravity of 1.010, which remains unchanged after water deprivation for 12 hours. She undergoes several courses of chemotherapy over the next year. During this time, the serum urea nitrogen level progressively increases. Which of the following abnormal laboratory findings is most likely to be reported for this patient? □ (A) Hepatitis B surface antigenemia □ (B) Hypercalcemia □ (C) Hypercholesterolemia □ (D) Hypergammaglobulinemia □ (E) Hyperglycemia □ (F) Hyperuricemia

Hypercalcemia

A 45-year-old man has had headaches, nausea, and vomiting that have worsened over the past 5 days. He has started "seeing spots" before his eyes. On physical examination, his blood pressure is 268/150 mm Hg. Urinalysis shows 1+ proteinuria; 2+ hematuria; and no glucose, ketones, or leukocytes. The serum urea nitrogen and creatinine levels are elevated. Which of the following histologic findings is most likely to be seen in this patient's kidneys? □ (A) Nodular glomerulosclerosis □ (B) Segmental tubular necrosis □ (C) Hyperplastic arteriolosclerosis □ (D) Mesangial IgA deposition □ (E) Glomerular crescents

Hyperplastic arteriolosclerosis

A 12-year-old boy has a history of episodes of severe abdominal and back pain since early childhood. On physical examination, he is afebrile, and there is no organomegaly. Laboratory studies show hemoglobin of 11.2 g/dL, platelet count of 194,000/mm3, and WBC count of 9020/mm3. The peripheral blood smear shows occasional sickled cells, nucleated RBCs, and Howell-Jolly bodies. Hemoglobin electrophoresis shows 1% hemoglobin A2, 6% hemoglobin F, and 93% hemoglobin S. Hydroxyurea therapy is found to be beneficial in this patient. Which of the following is the most likely basis for its therapeutic efficacy? □ (A) Increase in production of hemoglobin F □ (B) Increase in production of hemoglobin A □ (C) Decrease in overall globin synthesis □ (D) Stimulation of erythrocyte production □ (E) Increase in oxygen affinity of hemoglobin

Increase in production of hemoglobin F

A 29-year-old woman sees her physician because she has had a fever and sore throat for the past 3 days. On physical examination, her temperature is 38°C. The pharynx is erythematous, with yellowish tonsillar exudate. Group A Streptococcus pyogenes is cultured. She is treated with ampicillin and recovers fully in 7 days. Two weeks later, she develops fever and a rash, and notices a slight decrease in urinary output. Her temperature is 37.7°C, and there is a diffuse erythematous rash on the trunk and extremities. Urinalysis shows a pH of 6; specific gravity 1.022; 1+ proteinuria; 1+ hematuria; and no glucose or ketones. Microscopic examination of the urine shows RBCs and WBCs, including eosinophils, but no casts or crystals. What is the most likely cause of her disease? □ (A) Deposition of immune complexes with streptococcal antigens □ (B) Hematogenous dissemination of septic emboli □ (C) Renal tubular cell necrosis caused by bacterial toxins □ (D) Hypersensitivity reaction to ampicillin □ (E) Formation of antibodies against glomerular basement membrane

Hypersensitivity reaction to ampicillin

A 58-year-old woman dies of a cerebral infarction. Laboratory findings before death included serum urea nitrogen level of 110 mg/dL and creatinine level of 9.8 mg/dL. At autopsy, the kidneys are small (75 g) and have a coarsely granular surface appearance. Microscopic examination shows sclerotic glomeruli, a fibrotic interstitium, tubular atrophy, arterial thickening, and scattered lymphocytic infiltrates. Which of the following clinical findings was most likely reported on the patient's medical history? □ (A) Rash □ (B) Hypertension □ (C) Hemoptysis □ (D) Lens dislocation □ (E) Pharyngitis

Hypertension

A 5-year-old boy has a history of recurrent urinary tract infections. Urine cultures have grown Escherichia coli, Proteus mirabilis, and enterococcus. Physical examination now shows an abnormal constricted opening of the urethra on the ventral aspect of the penis, about 1.5 cm from the tip of the glans penis. There also is a cryptorchid testis on the right and an inguinal hernia on the left. What term best describes the child's penile abnormality? □ (A) Hypospadias □ (B) Phimosis □ (C) Balanitis □ (D) Epispadias □ (E) Bowen disease

Hypospadias

A 28-year-old, previously healthy man suddenly develops severe abdominal pain and begins passing red urine. There are no abnormalities on physical examination. Urinalysis shows a pH of 7; specific gravity 1.015; 1+ hematuria; and no protein, glucose, or ketones. The patient is given a device to use in straining the urine for calculi. The next day, the patient recovers a 0.3-cm stone that is sent for analysis. The chemical composition is found to be calcium oxalate. What underlying condition is most likely to be present? □ (A) Gout □ (B) Acute cystitis □ (C) Diabetes mellitus □ (D) Primary hyperparathyroidism □ (E) Idiopathic hypercalciuria

Idiopathic hypercalciuria

A 42-year-old woman has had nosebleeds, easy bruising, and increased bleeding with her menstrual periods for the past 4 months. On physical examination, her temperature is 37°C, pulse is 88/min, respirations are 18/min, and blood pressure is 90/60 mm Hg. She has scattered petechiae over the distal extremities. There is no organomegaly. Laboratory studies show hemoglobin of 12.3 g/dL, hematocrit of 37%, platelet count of 21,500/mm3, and WBC count of 7370/mm3. A bone marrow biopsy specimen shows a marked increase in megakaryocytes. The prothrombin and partial thromboplastin times are within the reference range. What is the most likely diagnosis? □ (A) Disseminated intravascular coagulation □ (B) Hemophilia B □ (C) Idiopathic thrombocytopenic purpura □ (D) Metastatic breast carcinoma □ (E) Thrombotic thrombocytopenic purpura □ (F) Vitamin K deficiency □ (G) Von Willebrand disease

Idiopathic thrombocytopenic purpura

A 3-year-old boy of Italian ancestry is brought to the physician because he has a poor appetite and is underweight for his age and height. Physical examination shows hepatosplenomegaly. The hemoglobin concentration is 6 g/dL, and the peripheral blood smear shows severely hypochromic and microcytic RBCs. The total serum iron level is normal, and the reticulocyte count is 10%. A radiograph of the skull shows maxillofacial deformities and an expanded marrow space. Which of the following is the most likely principal cause of this child's illness? □ (A) Reduced synthesis of hemoglobin F □ (B) Imbalance in production of α-globin and β-globin chains □ (C) Sequestration of iron in reticuloendothelial cells □ (D) Increased fragility of erythrocyte membrane □ (E) Relative deficiency of vitamin B12

Imbalance in production of α-globin and β-globin chains

A 12-year-old boy experienced sudden onset of severe abdominal pain and cramping accompanied by chest pain, nonproductive cough, and fever. On physical examination, his temperature is 39°C, pulse is 110/min, respirations are 22/min, and blood pressure is 80/50 mm Hg. He has diffuse abdominal tenderness, but no masses or organomegaly. Laboratory studies show a hematocrit of 18%. The peripheral blood smear is shown in the figure. A chest x-ray shows bilateral pulmonary infiltrates. Which of the following is the most likely mechanism for initiation of his pulmonary problems? □ (A) Intravascular hemolysis □ (B) Chronic hypoxia of the pulmonary parenchyma □ (C) Increased RBC adhesion to endothelium □ (D) Defects in the alternative pathway of complement activation □ (E) Formation of autoantibodies to alveolar basement membrane

Increased RBC adhesion to endothelium

A 23-year-old woman in her 25th week of pregnancy has felt no fetal movement for the past 3 days. Three weeks later, she still has not given birth and suddenly develops dyspnea with cyanosis. On physical examination, her temperature is 36.9°C, pulse is 102/min, respirations are 21/min, and blood pressure is 80/40 mm Hg. She has large ecchymoses over the skin of her entire body. A stool sample is positive for occult blood. Laboratory studies show an elevated prothrombin time and partial thromboplastin time. The platelet count is decreased, plasma fibrinogen is markedly decreased, and fibrin split products are detected. A blood culture is negative. Which of the following is the most likely cause of the bleeding diathesis? □ (A) Increased vascular fragility □ (B) Toxic injury to the endothelium □ (C) Reduced production of platelets □ (D) Increased consumption of clotting factors and platelets □ (E) Defects in platelet adhesion and aggregation

Increased consumption of clotting factors and platelets

A 27-year-old man visits the physician because he has had a fever for the past 10 days. On physical examination, his temperature is 37.9°C, pulse is 87/min, respirations are 21/min, and blood pressure is 100/55 mm Hg. A diastolic murmur is heard on auscultation of the chest. The tip of the spleen is palpable and tender. Laboratory studies show hemoglobin, 12.8 g/dL; hematocrit, 38.4%; platelet count, 231,000/mm3; and WBC count, 12,980/mm3 with 70% segmented neutrophils, 6% bands, 1% metamyelocytes, 19% lymphocytes, and 4% monocytes. The representative gross appearance of the spleen is shown in the figure. What is the most likely diagnosis? □ (A) Acute myelogenous leukemia □ (B) Disseminated histoplasmosis □ (C) Hodgkin lymphoma □ (D) Infective endocarditis □ (E) Metastatic carcinoma □ (F) Micronodular cirrhosis □ (G) Rheumatic heart disease

Infective endocarditis

A 26-year-old man has noted lumps in his neck that have been enlarging for the past 6 months. On physical examination, he has a group of enlarged, nontender right cervical lymph nodes. A biopsy of one of the lymph nodes shows scattered Reed-Sternberg cells, macrophages, lymphocytes, neutrophils, eosinophils, and a few plasma cells. Which of the following factors elaborated by the Reed-Sternberg cells has led to the appearance of the eosinophils within this lesion? □ (A) Platelet-derived growth factor □ (B) Cyclin D1 □ (C) Interleukin-5 □ (D) Trans-retinoic acid □ (E) Erythropoietin

Interleukin-5

A 38-year-old woman has experienced increasing dyspnea for the past 2 months. On physical examination, she is afebrile and normotensive. Inspiratory wheezes are noted on auscultation of the chest. A chest CT scan shows an 8 × 10 cm posterior mediastinal mass that impinges on the trachea and esophagus. A mediastinoscopy is performed, and the mass is biopsied. Histologically, there are scattered large multinucleated cells, with prominent nucleoli that mark with CD15, and lymphocytes and macrophages separated by dense collagenous bands. Which of the following is most likely to be seen microscopically in this biopsy specimen? □ (A) Atypical lymphocytes □ (B) Histiocytes with Birbeck granules □ (C) Hairy cells □ (D) Lacunar cells □ (E) Lymphoblasts □ (F) Myeloblasts

Lacunar cells

A 9-year-old boy is taken to his pediatrician because of a generalized seborrheic skin eruption and fever. He has been diagnosed and treated for otitis media several times in the past year. On physical examination, he has mild lymphadenopathy, hepatomegaly, and splenomegaly. The electron micrograph shown in the figure was taken from a mass lesion involving the mastoid bone. What is the most likely diagnosis? □ (A) Acute lymphoblastic leukemia □ (B) Multiple myeloma □ (C) Hodgkin lymphoma, mixed cellularity type □ (D) Langerhans cell histiocytosis □ (E) Disseminated tuberculosis

Langerhans cell histiocytosis

A 37-year-old woman visits her physician because of a cough and fever of 1 week's duration. On physical examination, her temperature is 38.3°C. She has diffuse crackles in all lung fields. A chest radiograph shows bilateral extensive infiltrates. CBC shows hemoglobin, 13.9 g/dL; hematocrit, 42%; MCV, 89 μm3; platelet count, 210,000/mm3; and WBC count, 56,000/mm3 with 63% segmented neutrophils, 15% bands, 6% metamyelocytes, 3% myelocytes, 1% blasts, 8% lymphocytes, 2% monocytes, and 2% eosinophils. The peripheral blood leukocyte alkaline phosphatase score is increased. Which of the following is the most likely diagnosis? □ (A) Chronic myelogenous leukemia □ (B) Hairy cell leukemia □ (C) Hodgkin lymphoma, lymphocyte depletion type □ (D) Leukemoid reaction □ (E) Acute lymphoblastic leukemia

Leukemoid reaction

A 35-year-old man has noticed bilateral breast enlargement over the past 6 months. On physical examination, both breasts are enlarged without masses. His right testis is 1.5 times larger than his left testis; both are firm and round. His serum estrogen is increased. An ultrasound scan shows a circumscribed 2-cm mass in the body of the right testis, and a right orchiectomy is performed. The mass has a grossly uniform, brown cut surface. On microscopic examination, the cells are large and round with granular eosinophilic cytoplasm along with rod-shaped crystalloids of Reinke. What is the most likely diagnosis? □ (A) Choriocarcinoma □ (B) Embryonal carcinoma □ (C) Gonadoblastoma □ (D) Leydig cell tumor □ (E) Seminoma □ (F) Teratoma □ (G) Yolk sac tumor

Leydig cell tumor

A 10-year-old child has experienced multiple episodes of pneumonia and meningitis with septicemia since infancy. Causative organisms that have been cultured include Streptococcus pneumoniae and Haemophilus influenzae. On physical examination, the child has no organomegaly and no deformities. Laboratory studies show hemoglobin of 9.2 g/dL, hematocrit of 27.8%, platelet count of 372,000/mm3, and WBC count of 10,300/mm3. A hemoglobin electrophoresis shows 1% hemoglobin A2, 7% hemoglobin F, and 92% hemoglobin S. Which of the following is the most likely cause of the repeated infections in this child? □ (A) Loss of normal splenic function from recurrent ischemic injury □ (B) Reduced synthesis of immunoglobulins □ (C) Impaired neutrophil production □ (D) Reduced synthesis of complement proteins by the liver □ (E) Reduced expression of adhesion molecules on endothelial cells

Loss of normal splenic function from recurrent ischemic injury

A 41-year-old man has had fevers with chills and rigors for the past 2 weeks. On physical examination, his temperature is 39.2°C. CBC shows hemoglobin, 13.9 g/dL; hematocrit, 40.5%; MCV, 93 μm3; platelet count, 210,000/mm3; and WBC count, 13,750/mm3 with 75% segmented neutrophils, 10% bands, 10% lymphocytes, and 5% monocytes. A bone marrow biopsy specimen shows hypercellularity with a marked increase in myeloid precursors at all stages of maturation and in band neutrophils. These findings are most likely caused by which of the following conditions? □ (A) Acute viral hepatitis □ (B) Glucocorticoid therapy □ (C) Lung abscess □ (D) Vigorous exercise □ (E) Acute myelogenous leukemia

Lung abscess

A 17 year old girl has had arthralgias and myalgias for several months. During the past week, she noticed a decreased output of urine, which is reddish-brown. On physical examination, her blood pressure is 160/100 mm Hg and she has an erythematous malar skin rash. The ANA and anti-double-stranded DNA test results are both positive. The serum urea nitrogen is 52 mg/dL. A renal biopsy is performed. The immunofluorescence for C3, C1q, IgG, IgA and IgM is positive; an example is shown below. EM is also shown below. Which of the following is the most likely diagnosis? (A) IgA nephropathy (B) Lupus nephritis (C) Renal cell carcinoma (D) Diabetic nephropathy (E) Chronic pyelonephritis

Lupus nephritis

A 14-year-old boy complains of a feeling of discomfort in his chest that has worsened over the past 5 days. On physical examination, he has generalized lymphadenopathy. A chest radiograph shows clear lung fields, but there appears to be widening of the mediastinum. A chest CT scan shows a 10-cm mass in the anterior mediastinum. A biopsy specimen of the mass shows lymphoid cells with lobulated nuclei having delicate, finely stippled, nuclear chromatin. There is scant cytoplasm, and many mitoses are seen. The cells express TdT, CD2, and CD7 antigens. Molecular analysis reveals a point mutation in the NOTCH1 gene. What is the most likely diagnosis? □ (A) Lymphoblastic lymphoma □ (B) Burkitt lymphoma □ (C) Hodgkin lymphoma, nodular sclerosing type □ (D) Mantle cell lymphoma □ (E) Follicular lymphoma □ (F) Small lymphocytic lymphoma

Lymphoblastic lymphoma

A 53-year-old woman has experienced nausea with vomiting and early satiety for the past 7 months. On physical examination, she is afebrile and has no lymphadenopathy or hepatosplenomegaly. CBC shows hemoglobin, 12.9 g/dL; hematocrit, 41.9%; platelet count, 263,000/mm3; and WBC count, 8430/mm3. An upper gastrointestinal endoscopy shows loss of the rugal folds of the stomach over a 4 × 8 cm area of the fundus. Gastric biopsy specimens reveal the presence of Helicobacter pylori organisms in the mucus overlying superficial epithelial cells. There are mucosal and submucosal monomorphous infiltrates of small lymphocytes, which are CD19+ and CD20+, but CD3−. After treatment of the H. pylori infection, her condition improves. What is the most likely diagnosis? □ (A) Acute lymphoblastic leukemia □ (B) Chronic lymphocytic leukemia □ (C) Diffuse large B-cell lymphoma □ (D) Follicular lymphoma □ (E) Hodgkin lymphoma, mixed cellularity type □ (F) MALT (marginal zone) lymphoma □ (G) Waldenström macroglobulinemia

MALT (marginal zone) lymphoma

A 45 year old man has experienced worsening headaches, nausea, and vomiting for the past five days. He has started "seeing spots" before his eyes. On physical examination, his blood pressure is 268/150 mm Hg. Urinalysis shows 1+ proteinuria, 2+ hematuria, and no glucose, ketones, or leukocytes. Serum urea nitrogen and creatinine are both elevated. Which of the following is the single best diagnosis? (A) Diabetic crisis (B) Sepsis and acute renal failure (C) Malignant hypertension (D) Berger's disease (E) Rapidly progressive glomerulonephritis

Malignant hypertension

A 49-year-old woman has experienced increasing weakness and chest pain over the past 6 months. On physical examination, she is afebrile and normotensive. Motor strength is 5/5 in all extremities, but diminishes to 4/5 with repetitive movement. There is no muscle pain or tenderness. Laboratory studies show hemoglobin, 14 g/dL; hematocrit, 42%; platelet count, 246,000/mm3; and WBC count, 6480/mm3. A chest CT scan shows an irregular 10 × 12 cm anterior mediastinal mass. The surgeon has difficulty removing the mass because it infiltrates surrounding structures. Microscopically, the mass is composed of large, spindled, atypical epithelial cells mixed with lymphoid cells. Which of the following is the most likely diagnosis of this lesion? □ (A) Granulomatous inflammation □ (B) Hodgkin lymphoma □ (C) Lymphoblastic lymphoma □ (D) Malignant thymoma □ (E) Metastatic breast carcinoma □ (F) Organizing abscess

Malignant thymoma

A 62-year-old man visits his physician because of prolonged fever and a 4-kg weight loss over the past 6 months. On physical examination, his temperature is 38.6°C. He has generalized nontender lymphadenopathy, and the spleen tip is palpable. Laboratory studies show hemoglobin, 10.1 g/dL; hematocrit, 30.3%; platelet count, 140,000/mm3; and WBC count, 24,500/mm3 with 10% segmented neutrophils, 1% bands, 86% lymphocytes, and 3% monocytes. A cervical lymph node biopsy specimen shows a nodular pattern of small lymphoid cells. A bone marrow specimen shows infiltrates of similar small cells having surface immunoglobulin that are CD5+, but CD10−. Cytogenetic analysis indicates t(11;14) in these cells. What is the most likely diagnosis? □ (A) Mantle cell lymphoma □ (B) Follicular lymphoma □ (C) Acute lymphoblastic leukemia □ (D) Burkitt lymphoma □ (E) Small lymphocytic lymphoma

Mantle cell lymphoma

A 60-year-old woman has had headaches and dizziness for the past 5 weeks. She has been taking cimetidine for heartburn and omeprazole for ulcers. On physical examination, she is afebrile and normotensive, and her face has a plethoric to cyanotic appearance. There is mild splenomegaly, but no other abnormal findings. Laboratory studies show hemoglobin, 21.7 g/dL; hematocrit, 65%; platelet count, 400,000/mm3; and WBC count, 30,000/mm3 with 85% polymorphonuclear leukocytes, 10% lymphocytes, and 5% monocytes. The peripheral blood smear shows abnormally large platelets and nucleated RBCs. The serum erythropoietin level is undetectable, but the ferritin level is normal. Which of the following is most characteristic of the natural history of this patient's disease? □ (A) Transformation into acute B lymphoblastic leukemia □ (B) Marrow fibrosis with extramedullary hematopoiesis □ (C) Spontaneous remissions and relapses without treatment □ (D) Increase in monoclonal serum immunoglobulin □ (E) Development of a gastric non-Hodgkin lymphoma

Marrow fibrosis with extramedullary hematopoiesis

A 78-year-old man complains of worsening malaise and fatigue over the past 5 months. On physical examination, he is afebrile and normotensive. The spleen tip is palpable. A CBC shows hemoglobin, 10.6 g/dL; hematocrit, 29.8%; MCV, 92 μm3; platelet count, 95,000/mm3; and WBC count, 4900/mm3 with 67% segmented neutrophils, 4% bands, 2% metamyelocytes, 22% lymphocytes, 5% monocytes, and 3 nucleated RBCs per 100 WBCs. The peripheral blood smear shows occasional teardrop cells. An examination of the bone marrow biopsy specimen and smear is most likely to show which of the following findings? □ (A) Marrow packed with myeloblasts □ (B) Marrow fibrosis with reduced hematopoiesis □ (C) Replacement of marrow by fat □ (D) Presence of numerous megaloblasts □ (E) Marked normoblastic erythroid hyperplasia

Marrow fibrosis with reduced hematopoiesis

A 31-year-old woman experiences abdominal pain and sees her physician 1 week later after noticing blood in her urine. She has had three episodes of urinary tract infection during the past year. There are no remarkable findings on physical examination. Urinalysis shows 2+ hematuria; 1+ proteinuria; hypercalciuria; and no glucose or ketones. Microscopic examination of the urine shows numerous RBCs and oxalate crystals. An intravenous pyelogram shows linear striations radiating into the renal papillae, along with small cystic collections of contrast material in dilated collecting ducts. She is advised to increase her daily intake of fluids, and her condition improves. Which of the following conditions is most likely to be associated with these findings? □ (A) Autosomal-dominant polycystic kidney disease □ (B) Gout □ (C) Medullary sponge kidney □ (D) Multicystic renal dysplasia □ (E) Autosomal-recessive polycystic kidney disease □ (F) Urothelial carcinoma □ (G) Vesicoureteral reflux

Medullary sponge kidney

A 30-year-old woman has had a constant feeling of lethargy since childhood. On physical examination, she is afebrile and has a pulse of 80/min, respirations of 15/min, and blood pressure of 110/70 mm Hg. The spleen tip is palpable, but there is no abdominal pain or tenderness. Laboratory studies show hemoglobin of 11.7 g/dL, platelet count of 159,000/mm3, and WBC count of 5390/mm3. The peripheral blood smear shows spherocytosis. The circulating RBCs show an increased osmotic fragility. An inherited abnormality in which of the following RBC components best accounts for these findings? □ (A) Glucose-6-phosphate dehydrogenase □ (B) Membrane cytoskeletal protein □ (C) α-Globin chain □ (D) Heme □ (E) β-Globin chain □ (F) Carbonic anhydrase

Membrane cytoskeletal protein

A 38-year-old woman sees her physician because she has been feeling tired and lethargic for several months. On physical examination, she is afebrile, and her blood pressure is 140/90 mm Hg. Laboratory findings show hemoglobin, 10.3 g/dL; hematocrit, 30.9%; platelet count, 310,700/mm3; and WBC count, 5585/mm3. The serum creatinine level is 5.8 mg/dL. C3 nephritic factor is present in serum, and the ANA test result is negative. Urinalysis shows 2+ proteinuria. A renal biopsy is done; microscopic examination shows hypercellular glomeruli and prominent electron-dense deposits along the lamina densa of the glomerular basement membrane. Which of the following forms of glomerulonephritis is most likely to be present in this patient? □ (A) Postinfectious glomerulonephritis □ (B) Rapidly progressive glomerulonephritis □ (C) Membranoproliferative glomerulonephritis □ (D) Chronic glomerulonephritis □ (E) Membranous glomerulonephritis

Membranoproliferative glomerulonephritis

A 62 year-old man has mild generalized edema. He was previously found to have 7 grams of protein in his urine over a 24 hr period. Renal biopsy reveals thick glomerular basement membranes by light microscopy (shown below) and numerous subepithelial glomerular basement membrane deposits by electron microscopy. Which of the following is the best diagnosis of this patient's clinical and pathologic findings? (A) Diabetic nephrosclerosis (B) Focal segmental glomerulosclerosis (C) Membranoproliferative glomerulonephritis type I (D) Membranous glomerulonephritis (E) Minimal change disease

Membranous glomerulonephritis

A 25-year-old man has a 5-year history of celiac sprue. Several days after a mild upper respiratory infection, he begins passing dark red-brown urine. The dark urine persists for the next 3 days and then becomes clear and yellow, only to become red-brown again 1 month later. There are no remarkable findings on physical examination. Urinalysis shows a pH of 6.5; specific gravity 1.018; 3+ hematuria; 1+ proteinuria; and no glucose or ketones. Microscopic examination of the urine shows RBCs and no WBCs, casts, or crystals. A 24-hour urine protein level is 200 mg. A renal biopsy specimen from the glomeruli of this patient is most likely to show which of the following alterations? □ (A) Subepithelial electron-dense deposits □ (B) Granular staining of the basement membrane by anti-IgG antibodies □ (C) Mesangial IgA staining by immunofluorescence □ (D) Diffuse proliferation and basement membrane thickening □ (E) Thrombosis in the glomerular capillaries

Mesangial IgA staining by immunofluorescence

A 31 year old woman is evaluated two days after passing a kidney stone. For the past two years she has had symptoms of dry eyes, dry mouth, and Reynaud's phenomenon (tips of fingers turn cyanotic in cold). Arterial blood gas shows pH 7.36, pCO2 35. Her chem 7 is: Na 142, K 2.9, Cl 112, HCO3 20, BUN 15, Cr 1.2, Glucose 95. What is the primary acid base disorder? (A) Metabolic acidosis (B) Metabolic alkalosis (C) Respiratory acidosis (D) Respiratory alkalosis

Metabolic acidosis

A 50 year old man with acute diarrhea after eating oysters 4 hr ago presents with an arterial blood gas of pH = 7.23/pCO2 = 23. On the Chem 7, serum HCO3- = 10. He weighs 70 kg. What is the primary acid base disorder? (A) Metabolic acidosis (B) Metabolic alkalosis (C) Respiratory acidosis (D) Respiratory alkalosis

Metabolic acidosis

A 45-year-old man has a 3-day history of flank pain and fever. On physical examination, his temperature is 37.9°C. There is right costovertebral angle tenderness. Laboratory studies include a urine culture that is positive for Escherichia coli. The WBC count is 13,310/mm3. Two days later, he becomes hypotensive, and a blood culture is positive for E. coli. He requires increasing pressor support to maintain blood pressure. He develops a guaiac-positive stool and ecchymoses of the skin. CBC shows hemoglobin of 9.2 g/dL, hematocrit of 28.1%, and platelet count of 70,000/mm3. Increased amounts of fibrin split products are identified in the blood (elevated D dimer). Which of the following conditions is most likely responsible for the low hematocrit? □ (A) Warm autoimmune hemolytic anemia □ (B) Paroxysmal nocturnal hemoglobinuria □ (C) Microangiopathic hemolytic anemia □ (D) β-Thalassemia major □ (E) Aplastic anemia

Microangiopathic hemolytic anemia

A 3 year old, previously healthy boy has become increasingly lethargic during the past three weeks. On physical examination, his temperature is 36.9ºC, pulse 89/min, respirations 15/min, and blood pressure 140/90 mm Hg. Periorbital edema is present. Urinalysis shows 4+ proteinuria and no blood, ketones, or glucose. Which of the following is the most likely diagnosis? (A) Minimal change disease (B) Henoch-Schölein disease (C) Acute pyelonephritis (D) Rapidly progressive glomerulonephritis (E) IgA nephropathy

Minimal change disease

A 30-year-old man visits his physician because he has noticed increasing enlargement and a feeling of heaviness in his scrotum for the past year. On physical examination, the right testis is twice its normal size, and it is firm and slightly tender. An ultrasound examination shows a 3.5-cm solid mass in the right testis. Abdominal CT scan shows enlargement of the para-aortic lymph nodes. Multiple lung nodules are seen on a chest radiograph. Laboratory findings include markedly increased serum levels of chorionic gonadotropin and α-fetoprotein. Which of the following testicular neoplasms is the most likely diagnosis? □ (A) Leydig cell tumor □ (B) Mixed germ cell tumor □ (C) Pure spermatocytic seminoma □ (D) Choriocarcinoma □ (E) Metastatic adenocarcinoma of the prostate gland □ (F) Large diffuse B-cell lymphoma

Mixed germ cell tumor

A 71-year-old, currently healthy man visits his physician for a checkup because he is worried about his family history of prostate cancer. Physical examination does not indicate any abnormalities. Because of the patient's age and family history, his prostate-specific antigen (PSA) level is immediately measured, and the PSA level is 5 ng/mL. Six months later, the PSA level is 6 ng/mL. A urologist obtains transrectal biopsy specimens, and microscopic examination shows multifocal areas of prostatic intraepithelial neoplasia and glandular hyperplasia. Based on these findings, what is the most appropriate course of management for this patient? □ (A) Antibiotic therapy □ (B) Monitoring PSA levels □ (C) Multiagent chemotherapy □ (D) Radiation therapy □ (E) Radical prostatectomy □ (F) Transurethral prostate resection

Monitoring PSA levels

A 67-year-old man has had increasing weakness, fatigue, and weight loss over the past 5 months. He now has decreasing vision in both eyes and has headaches and dizziness. His hands are sensitive to cold. On physical examination, he has generalized lymphadenopathy and hepatosplenomegaly. Laboratory studies indicate hyperproteinemia with a serum protein level of 15.5 g/dL and albumin concentration of 3.2 g/dL. A bone marrow biopsy is performed, and microscopic examination of the specimen shows infiltration of small plasmacytoid lymphoid cells with Russell bodies in the cytoplasm. Which of the following findings is most likely to be reported for this patient? □ (A) Monoclonal IgM spike in serum □ (B) WBC count of 255,000/mm3 □ (C) Hypercalcemia □ (D) Bence Jones proteinuria □ (E) Karyotype with t(14;18) translocation

Monoclonal IgM spike in serum

A 48-year old man visits his physician for a routine health maintenance examination. He has no complaints other than worrying about getting older and having cancer. Physical examination shows that he is afebrile and normotensive. There is no hepatosplenomegaly or lymphadenopathy. Laboratory studies show a total serum protein level of 7.4 g/dL and albumin level of 3.9 g/dL. Serum calcium and phosphorus levels are normal. Urinalysis shows no Bence Jones proteinuria. Hemoglobin is 13.6 g/dL, platelet count is 301,500/mm3, and WBC count is 6630/mm3. A serum protein electrophoresis shows a small (2.8-g) spike of γ-globulin, which is determined by immunoelectrophoresis to be IgG kappa. A bone marrow biopsy specimen shows normal cellularity with maturation of all cell lines. Plasma cells constitute about 4% of the marrow. A bone scan is normal, and there are no areas of increased uptake. What is the most likely diagnosis? □ (A) Solitary plasmacytoma □ (B) Waldenström macroglobulinemia □ (C) Monoclonal gammopathy of undetermined significance □ (D) Heavy-chain disease □ (E) Multiple myeloma □ (F) Reactive systemic amyloidosis

Monoclonal gammopathy of undetermined significance

A 23-year-old woman has noticed that she develops a skin rash if she spends prolonged periods outdoors. She has a malar skin rash on physical examination. Laboratory studies include a positive ANA test result with a titer of 1 : 1024 and a "rim" pattern. An anti-double-stranded DNA test result also is positive. The hemoglobin concentration is 12.1 g/dL, hematocrit is 35.5%, MCV is 89 μm3, platelet count is 109,000/mm3, and WBC count is 4500/mm3. Which of the following findings is most likely to be shown by a WBC differential count? □ (A) Eosinophilia □ (B) Thrombocytosis □ (C) Monocytosis □ (D) Neutrophilia □ (E) Basophilia

Monocytosis

A 63-year-old man has noted increasing back pain for 7 months. He has had three respiratory tract infections with Streptococcus pneumoniae within the past year. On examination, he has pitting edema to his thighs. Laboratory studies show total serum protein, 9.6 g/dL; albumin, 3.5 g/dL; creatinine, 3 mg/dL; urea nitrogen, 28 mg/dL; and glucose, 79 mg/dL. Urinalysis shows proteinuria of 4 g/24 hr, but no glucosuria or hematuria. Abdominal CT scan shows enlarged kidneys without cysts or masses. A renal biopsy specimen shows deposits of amorphous pink material within glomeruli, interstitium, and arteries with H&E stain. Which of the following diseases is he most likely to have? □ (A) Analgesic nephropathy □ (B) Diabetes mellitus □ (C) Membranous glomerulonephritis □ (D) Multiple myeloma □ (E) Systemic lupus erythematosus □ (F) Wegener granulomatosis

Multiple myeloma

A 79-year-old man has had increasing back pain and fatigue for the past 6 months. On physical examination, there are no remarkable findings. Laboratory studies include a CBC with hemoglobin of 9.6 g/dL, platelet count of 241,600/mm3, and WBC count of 7160/mm3. The serum total protein is 9.8 g/dL, albumin is 3.6 g/dL, glucose is 72 mg/dL, creatinine is 3.3 mg/dL, and urea nitrogen is 30 mg/dL. A dipstick urinalysis shows a pH of 7; specific gravity 1.011; and no blood, protein, or glucose. One month later, he develops a cough with fever, and Streptococcus pneumoniae is cultured from his sputum. Despite antibiotic therapy, he develops sepsis and dies. At autopsy, the kidneys are normal in size, but microscopic examination shows dilated tubules filled with amorphous blue-to-pink casts and occasional multinucleated giant cells. What is the most likely underlying cause of this patient's death? □ (A) Cystinuria □ (B) Diabetes mellitus □ (C) Gout □ (D) Multiple myeloma □ (E) Parathyroid adenoma □ (F) Systemic lupus erythematosus

Multiple myeloma

A 35-year-old man and his 33-year-old wife are childless. They have tried to conceive a child for 12 years, and now they undergo an infertility work-up. On physical examination, neither spouse has any remarkable findings. Laboratory studies show that the man has a sperm count in the low-normal range. On microscopic examination of the seminal fluid, the sperm have a normal morphologic appearance. A testicular biopsy is done. The biopsy specimen shows patchy atrophy of seminiferous tubules, but the remaining tubules show active spermatogenesis. Which of the following disorders is the most likely cause of these findings? □ (A) Mumps virus infection □ (B) Cryptorchidism □ (C) Hydrocele □ (D) Klinefelter syndrome □ (E) Prior chemotherapy

Mumps virus infection

A 26-year-old man has had a fever with nonproductive cough for the past 10 weeks. On examination, his temperature is 37.4°C. A chest radiograph shows a 4-cm left upper lobe nodule. CBC shows hemoglobin, 13.3 g/dL; hematocrit, 40.5%; platelet count, 281,000/mm3; and WBC count, 13,760/mm3 with 38% segmented neutrophils, 2% bands, 45% lymphocytes, and 15% monocytes. What is the most likely diagnosis? □ (A) Acute lymphoblastic leukemia/lymphoma □ (B) Hodgkin lymphoma, lymphocyte rich type □ (C) Mycobacterium tuberculosis granuloma □ (D) Myelodysplastic syndrome □ (E) Staphylococcus aureus abscess

Mycobacterium tuberculosis granuloma

A 68 year man is seen for his annual physical exam, which is normal. However, laboratory studies reveal Hct 35%, WBC count 2500/mm3, platelet count 90,000/mm3, and an MCV of 103 fL. After review of his records, it is found that studies in previous years were normal. A bone marrow aspirate and peripheral blood smear are obtained and the results are shown below. Which of the following is the most likely diagnosis? (A) Hodgkin's disease, nodular sclerosis type (B) Acute lymphadenitis (C) Follicular lymphoma (D) Chronic myelogenous leukemia (E) Myelodysplasia

Myelodysplasia

A 50-year-old man was diagnosed with a diffuse large B-cell lymphoma. He underwent intensive chemotherapy, and a complete remission was achieved for 7 years. He now reports fatigue and recurrent pulmonary and urinary tract infections over the past 4 months. Physical examination shows no masses, lymphadenopathy, or hepatosplenomegaly. CBC shows hemoglobin, 8.7 g/dL; hematocrit, 25.2%; MCV, 88 μm3; platelet count, 67,000/mm3; and WBC count, 2300/mm3 with 15% segmented neutrophils, 5% bands, 2% metamyelocytes, 2% myelocytes, 6% myeloblasts, 33% lymphocytes, 35% monocytes, and 2% eosinophils. A bone marrow biopsy specimen shows 90% cellularity with many immature cells, including ringed sideroblasts, megaloblasts, hypolobated megakaryocytes, and myeloblasts. Karyotypic analysis shows 5q deletions in many cells. Which of the following is most likely now to have occurred in this patient? □ (A) Relapse of his previous lymphoma □ (B) Transformation of lymphoma into myeloid leukemia □ (C) Myelodysplasia related to therapy for the previous tumor □ (D) De novo acute myeloblastic leukemia □ (E) Myeloid metaplasia with myelofibrosis

Myelodysplasia related to therapy for the previous tumor

A 77-year-old man has experienced increasing malaise and a 6-kg weight loss over the past year. He has noted more severe and constant back pain for the past 3 months. On physical examination, his temperature is 38.7°C. His prostate is firm and irregular when palpated on digital rectal examination. There is no organomegaly. A stool sample is negative for occult blood. Laboratory studies include a urine culture positive for Escherichia coli, serum glucose of 70 mg/dL, creatinine of 1.1 mg/dL, total bilirubin of 1 mg/dL, alkaline phosphatase of 293 U/L, calcium of 10.3 mg/dL, phosphorus of 2.6 mg/dL, and PSA of 25 ng/mL. CBC shows hemoglobin, 9.1 g/dL; hematocrit, 27.3%; MCV, 94 μm3; platelet count, 55,600/mm3; and WBC count, 3570/mm3 with 18% segmented neutrophils, 7% bands, 2% metamyelocytes, 1% myelocytes, 61% lymphocytes, 11% monocytes, and 3 nucleated RBCs per 100 WBCs. What is the most likely diagnosis? □ (A) Anemia of chronic disease □ (B) Aplastic anemia □ (C) Hemolytic anemia □ (D) Megaloblastic anemia □ (E) Myelophthisic anemia □ (F) Thalassemia

Myelophthisic anemia

Over the past 72 hours, a 44-year-old man has experienced worsening headache, nausea, and vomiting. On physical examination, his blood pressure is 276/158 mm Hg, and there is bilateral papilledema. Urinalysis shows 2+ proteinuria; 1+ hematuria; and no glucose or ketones. Which of the following renal lesions is most likely to be present in this patient? □ (A) Papillary necrosis □ (B) Acute infarction □ (C) Necrotizing arteriolitis □ (D) Acute tubular necrosis □ (E) Acute pyelonephritis

Necrotizing arteriolitis

A 53-year-old woman has had dysuria and urinary frequency for the past week. On physical examination, her temperature is 38°C, and she has pain on palpation over the left costovertebral angle. Laboratory findings show glucose, 177 mg/dL; hemoglobin A1c, 9.8%; hemoglobin, 13.1 g/dL; platelet count, 232,200/mm3; and WBC count, 11,320/mm3. Urinalysis shows a pH of 6.5; specific gravity 1.016; 2+ glucosuria; and no blood, protein, or ketones. Microscopic examination of the urine shows numerous neutrophils, and a urine culture is positive for Escherichia coli. Which of the following complications is most likely to develop in this patient? □ (A) Acute tubular necrosis □ (B) Necrotizing papillitis □ (C) Crescentic glomerulonephritis □ (D) Hydronephrosis □ (E) Renal calculi

Necrotizing papillitis

Several members of a family developed chronic renal failure by age 50 years. Most are males. The affected individuals also developed visual problems. Some younger family members have proteinuria and hematuria on urinalysis. A renal biopsy specimen from a 20-year-old man shows prominent tubular foam cells and glomerular basement membrane thickening and thinning. Family members with this disease are most likely to have which of the following additional manifestations? □ (A) Watery diarrhea □ (B) Nerve deafness □ (C) Presenile dementia □ (D) Dilated cardiomyopathy □ (E) Infertility

Nerve deafness

A clinical study is performed to assess outcomes in patients who have macrocytic anemias. A comparison of laboratory testing strategies shows that the best strategy includes testing for vitamin B12 (cobalamin) and folate. What is the most important reason for ordering these tests simultaneously? □ (A) Both nutrients are absorbed similarly □ (B) Therapy for one deficiency also treats the other □ (C) The peripheral blood smear appears the same for both deficiencies □ (D) Aplastic anemia can result from lack of either nutrient □ (E) Neurologic injury must be avoided

Neurologic injury must be avoided

A 55-year-old woman has had poorly controlled hyperglycemia for many years. She sees her physician after experiencing burning pain on urination for 3 days. Physical examination shows a 2-cm ulceration on the skin of the heel and reduced sensation in the lower extremities. Her visual acuity is 20/100 bilaterally. Urinalysis shows 1+ proteinuria; 2+ glucosuria; and no blood, ketones, or urobilinogen. A urine culture contains more than 100,000 colony-forming units/mL of Klebsiella pneumoniae. Which of the following pathologic findings is most likely to be present in both kidneys? □ (A) Deposits of IgG and C3 in the glomerular basement membrane □ (B) Effacement of podocyte foot processes □ (C) Glomerular crescents □ (D) Mesangial deposits of IgA □ (E) Necrotizing granulomatous vasculitis □ (F) Nodular hyaline mesangial masses □ (G) Thickening and thinning of the glomerular basement membrane

Nodular hyaline mesangial masses

A clinical trial of two pharmacologic agents compares one agent that inhibits 5α-reductase and diminishes dihydrotestosterone (DHT) synthesis in the prostate with another agent that acts as an α1-adrenergic receptor. The subjects are 40 to 80 years old. The study will determine whether symptoms of prostate disease are ameliorated in the individuals who take these drugs. Which of the following diseases of the prostate is most likely to benefit from one or both of these drugs? □ (A) Acute prostatitis □ (B) Adenocarcinoma □ (C) Leiomyoma □ (D) Chronic prostatitis □ (E) Nodular hyperplasia

Nodular hyperplasia

A 33-year-old woman reports having generalized fatigue and night sweats for 3 months. Physical examination shows nontender right cervical lymphadenopathy. Biopsy of one lymph node shows a microscopic pattern of thick bands of fibrous connective tissue with intervening lymphocytes, plasma cells, eosinophils, macrophages, and occasional Reed- Sternberg cells. An abdominal CT scan and bone marrow biopsy specimen show no abnormalities. Which of the following is the most likely subtype and stage of this patient's disease? □ (A) Lymphocyte predominance, stage I □ (B) Lymphocyte predominance, stage II □ (C) Nodular sclerosis, stage I □ (D) Mixed cellularity, stage II □ (E) Lymphocyte depletion, stage III

Nodular sclerosis, stage I

A 62-year-old man goes to the emergency department in an obvious state of inebriation. He is well known there because this scenario has been repeated many times over the years. On physical examination, he is afebrile. The spleen tip is palpable, and the liver edge is firm. Laboratory studies show hemoglobin of 8.2 g/dL, hematocrit of 25.1%, MCV of 107 μm3, platelet count of 135,000/mm3, and WBC count of 3920/mm3. The peripheral blood smear shows prominent anisocytosis and macrocytosis. Polychromatophilic RBCs are difficult to find. A few of the neutrophils show six to seven nuclear lobes. Which of the following is the most likely explanation of these findings? □ (A) Mechanical fragmentation of RBCs □ (B) Increased susceptibility to lysis by complement □ (C) Nuclear maturation defects resulting from impaired DNA synthesis □ (D) Hemolysis of antibody-coated cells □ (E) Reduced deformability of the RBC membrane □ (F) Production of abnormal hemoglobin □ (G) Imbalance in synthesis of α-globin and β-globin chains

Nuclear maturation defects resulting from impaired DNA synthesis

A 61-year-old man reports a history of back pain for 5 months. He has recently developed a cough that is productive of yellow sputum. On physical examination, he is febrile, and diffuse rales are heard on auscultation of the lungs. He has no lymphadenopathy or splenomegaly. Laboratory studies include a sputum culture that grew Streptococcus pneumoniae. The serum creatinine level is 3.7 mg/dL, and the urea nitrogen level is 35 mg/dL. The figure shows a skull radiograph. During his hospitalization, a bone marrow biopsy is performed. Which of the following is the biopsy specimen most likely to show? □ (A) Scattered small granulomas □ (B) Numerous plasma cells □ (C) Nodules of small mature lymphocytes □ (D) Occasional Reed-Sternberg cells □ (E) Hypercellularity with many blasts

Numerous plasma cells

A 25-year-old, previously healthy man suddenly develops severe pain in the scrotum. The pain continues unabated for 6 hours, and he goes to the emergency department. On physical examination, he is afebrile. There is exquisite tenderness of a slightly enlarged right testis, but there are no other remarkable findings. The gross appearance of the right testis is shown in the figure. Which of the following conditions is most likely to cause these findings? □ (A) Disseminated tuberculosis □ (B) Invasive germ cell tumor □ (C) Lymphedema □ (D) Obstruction of blood flow □ (E) Previous vasectomy

Obstruction of blood flow

A 73-year-old man has been healthy all his life. He takes no medications and has had no major illnesses or surgeries. For the past year, he has become increasingly tired and listless, and he appears pale. Physical examination shows no hepatosplenomegaly and no deformities. CBC shows hemoglobin, 9.7 g/dL; hematocrit, 29.9%; MCV, 69.7 mm3; RBC count, 4.28 million/mm3; platelet count, 331,000/mm3; and WBC count, 5500/mm3. Which of the following is the most likely underlying condition causing this patient's findings? □ (A) Occult malignancy □ (B) Autoimmune hemolytic anemia □ (C) β-Thalassemia major □ (D) Chronic alcoholism □ (E) Vitamin B12 deficiency □ (F) Hemophilia A

Occult malignancy

A 15-year-old boy visits his physician because of high fever of 10 days' duration. Physical examination shows a temperature of 38°C. He has scattered petechial hemorrhages on the trunk and extremities. There is no enlargement of liver, spleen, or lymph nodes. The CBC shows hemoglobin, 13.2 g/dL; hematocrit, 38.9%; MCV, 93 μm3; platelet count, 175,000/mm3; and WBC count, 1850/mm3 with 1% segmented neutrophils, 98% lymphocytes, and 1% monocytes. Bone marrow biopsy examination does not show any abnormal cells. Which of the following is the most likely diagnosis? □ (A) Acute lymphoblastic leukemia □ (B) Acute myelogenous leukemia □ (C) Aplastic anemia □ (D) Idiopathic thrombocytopenic purpura □ (E) Overwhelming bacterial infection

Overwhelming bacterial infection

A 30-year-old, previously healthy man passes dark brown urine several days after starting the prophylactic antimalarial drug primaquine. On physical examination, he appears pale and is afebrile. There is no organomegaly. Laboratory studies show that his serum haptoglobin level is decreased. Which of the following is the most likely explanation of these findings? □ (A) Mechanical fragmentation of RBCs □ (B) Increased susceptibility to lysis by complement □ (C) Nuclear maturation defects resulting from impaired DNA synthesis □ (D) Impaired globin synthesis □ (E) Hemolysis of antibody-coated cells □ (F) Oxidative injury to hemoglobin □ (G) Reduced deformability of the RBC membrane

Oxidative injury to hemoglobin

The top of the diaper is often noted to be damp on a girl infant. Radiologic imaging with contrast enhancement shows that there is a connection from the bladder to umbilicus. What is the most likely diagnosis? □ (A) Congenital diverticulum □ (B) Exstrophy □ (C) Persistent urachus □ (D) Vesicoureteral reflux □ (E) Vitelline duct remnant

Persistent urachus

A 19-year-old man comes to his physician complaining of worsening local pain and irritation with difficult urination over the past 3 years. He has become more sexually active during the past year and describes his erections as painful. Physical examination shows that he is not circumcised. The prepuce (foreskin) cannot be easily retracted over the glans penis. What is the most likely diagnosis? □ (A) Epispadias □ (B) Bowenoid papulosis □ (C) Phimosis □ (D) Genital candidiasis □ (E) Paraphimosis

Phimosis

A 24 year old Nepalese man presents with high fever, substantial weight loss, swelling of the spleen and liver, and anemia. He is diagnosed with Kala azar or visceral leishmaniasis. You advise him that he most likely contracted his disease from (A) Culicine mosquitoes (B) Phlebotomus sandflies (C) Soft ticks (D) A brothel in Northern India

Phlebotomus sandflies

A 55-year-old, otherwise healthy man has experienced minor fatigue on exertion for the past 9 months. He has no significant previous medical or surgical history. On physical examination, there are no remarkable findings. Laboratory studies show hemoglobin of 11.7 g/dL, hematocrit of 34.8%, MCV of 73 μm3, platelet count of 315,000/mm3, and WBC count of 8035/mm3. Which of the following is the most sensitive and cost-effective test that the physician should order to help to determine the cause of these findings? □ (A) Serum iron □ (B) Serum transferrin □ (C) Serum haptoglobin □ (D) Bone marrow biopsy □ (E) Serum ferritin □ (F) Hemoglobin electrophoresis

Serum ferritin

A 17-year-old boy reports passage of dark urine to his physician. He has a history of multiple bacterial infections and venous thromboses for the past 10 years, including portal vein thrombosis in the previous year. On physical examination, his right leg is swollen and tender. CBC shows hemoglobin, 9.8 g/dL; hematocrit, 29.9%; MCV, 92 μm3; platelet count, 150,000/mm3; and WBC count, 3800/mm3 with 24% segmented neutrophils, 1% bands, 64% lymphocytes, 10% monocytes, and 1% eosinophils. He has a reticulocytosis, and his serum haptoglobin level is very low. A mutation affecting which of the following gene products is most likely to give rise to this clinical condition? □ (A) Spectrin □ (B) Glucose-6-phosphate dehydrogenase □ (C) Phosphatidylinositol glycan A (PIGA) □ (D) β-Globin chain □ (E) Factor V □ (F) Prothrombin G20210A

Phosphatidylinositol glycan A (PIGA)

A 26-year-old woman has experienced chronic fatigue since early childhood. She also has had episodes of severe pain in the abdomen, back, and legs. On physical examination, there is no organomegaly. Laboratory studies show hemoglobin, 8.9 g/dL; hematocrit, 26.9%; platelet count, 300,100/mm3; and WBC count, 5560/mm3. Hemoglobin electrophoresis shows 1% hemoglobin A2, 6% hemoglobin F, and 93% hemoglobin S. Which of the following is this patient most likely to develop as a complication of the underlying disease? □ (A) Micronodular cirrhosis □ (B) Chronic atrophic gastritis □ (C) Pigment gallstones □ (D) High rate of stillbirths □ (E) Esophageal web

Pigment gallstones

In an epidemiologic study of anemias, the findings show that there is an increased prevalence of anemia in individuals of West African ancestry. By hemoglobin electrophoresis, a subset of individuals of this ancestry are found to have increased hemoglobin S levels. The distribution of infectious illnesses is correlated with the prevalence of hemoglobin S in this population. Which of the following infectious agents is most likely to account for these observations? □ (A) Cryptococcus neoformans □ (B) Borrelia burgdorferi □ (C) Treponema pallidum □ (D) Plasmodium falciparum □ (E) Clostridium perfringens □ (F) Trypanosoma gambiense □ (G) Schistosoma haematobium

Plasmodium falciparum

Which of the following bloodborne pathogens is the most common cause of death in humans? (A) Babesia microti (B) Plasmodium vivax (C) Plasmodium ovale (D) Plasmodium ovale (E) Plasmodium falciparum

Plasmodium falciparum

A 22-year-old woman has experienced febrile episodes over the past 2 weeks. On physical examination, her temperature is 37.5°C, pulse is 82/min, respirations are 18/min, and blood pressure is 105/65 mm Hg. Laboratory studies show hemoglobin of 10.8 g/dL, hematocrit of 32.5%, platelet count of 245,700/mm3, and WBC count of 8320/mm3. The serum haptoglobin level is decreased, but direct and indirect Coombs test results are negative. The reticulocyte count is increased. The prothrombin time is 12 seconds, and the partial thromboplastin time is 31 seconds. The patient is observed over the next week and found to have temperature spikes to 39.1°C, with shaking chills every 48 hours. Infection with which of the following organisms is most likely to cause this patient's illness? □ (A) Aspergillus niger □ (B) Babesia microti □ (C) Dirofilaria □ (D) Escherichia coli □ (E) Plasmodium vivax □ (F) Wuchereria bancrofti

Plasmodium vivax

A 61 year old woman presents to her physician because she has experienced increasing malaise for the past five years. On physical examination, there are no abnormalities other than a blood pressure of 150/95 mm Hg. One week later, she dies suddenly and unexpectedly. At autopsy, both kidneys have the external appearance shown below. Which of the following is the most likely diagnosis? (A) Renal cell carcinoma (B) Polycystic kidney disease (C) Acute tubular necrosis (D) Chronic pyelonephritis (E) Diabetic nephropathy

Polycystic kidney disease

A 50-year-old man has had headache, dizziness, and fatigue for the past 3 months. His friends have been commenting about his increasingly ruddy complexion. He also has experienced generalized and severe pruritus, particularly when showering. He notes that his stools are dark. On physical examination, he is afebrile, and his blood pressure is 165/90 mm Hg. There is no hepatosplenomegaly or lymphadenopathy. A stool sample is positive for occult blood. CBC shows hemoglobin, 22.3 g/dL; hematocrit, 67.1%; MCV, 94 μm3; platelet count, 453,000/mm3; and WBC count, 7800/mm3. The serum erythropoietin level is very low. What is the most likely diagnosis? □ (A) Myelodysplastic syndrome □ (B) Essential thrombocytosis □ (C) Chronic myelogenous leukemia □ (D) Erythroleukemia □ (E) Polycythemia vera

Polycythemia vera

A 45-year-old woman has experienced worsening arthritis of her hands and feet for the past 15 years. On physical examination, there are marked deformities of the hands and feet, with ulnar deviation of the hands and swan-neck deformities of the fingers. Laboratory studies show an elevated level of rheumatoid factor. CBC shows hemoglobin, 11.6 g/dL; hematocrit, 34.8%; MCV, 87 μm3; platelet count, 268,000/mm3; and WBC count, 6800/mm3. There is a normal serum haptoglobin level, serum iron concentration of 20 μg/dL, total iron-binding capacity of 195 μg/dL, percent saturation of 10.2, and serum ferritin concentration of 317 ng/mL. No fibrin split products are detected. The reticulocyte concentration is 1.1%. What is the most likely mechanism underlying this patient's hematologic abnormalities? □ (A) Poor use of stored iron □ (B) Space-occupying lesions in the bone marrow □ (C) Mutation in the phosphatidylinositol glycan A (PIGA) gene □ (D) Sequestration of red blood cells in splenic sinusoids □ (E) Impaired synthesis of β-globin chains □ (F) Warm antibodies against red blood cell membranes

Poor use of stored iron

A 25-year-old woman experiences sudden onset of fever, malaise, and nausea. On physical examination, her temperature is 38.2°C, pulse is 85/min, respirations are 18/min, and blood pressure is 140/90 mm Hg. A routine urinalysis shows 1+ proteinuria, 4+ hematuria, and no ketones or glucose. RBC casts are seen on microscopic examination of the urine. A renal biopsy is performed, and light microscopic examination shows marked glomerular hypercellularity with neutrophils in glomerular capillary loops. Immunofluorescence microscopy shows granular deposition of IgG and C3 in glomerular capillary basement membranes. Electron microscopy shows electron-dense subepithelial "humps." What is the most likely diagnosis? □ (A) Goodpasture syndrome □ (B) Systemic amyloidosis □ (C) Membranous glomerulonephritis □ (D) Diabetes mellitus □ (E) Postinfectious glomerulonephritis

Postinfectious glomerulonephritis

A 45-year-old woman has experienced episodes of blurred vision and headaches for the past 6 months. She has had worsening confusion with paresthesias over the past 3 days. On physical examination, she has a temperature of 39.6°C, pulse of 100/min, respiratory rate of 20/min, and blood pressure of 80/50 mm Hg. Petechial hemorrhages are noted over her trunk and extremities. Laboratory findings include hemoglobin, 10.9 g/dL; hematocrit, 34%; MCV, 96/min3; platelet count, 28,000/mm3; and WBC count, 8500/mm3. Fragmented RBCs are noted on her peripheral blood smear. Blood urea nitrogen is 40 mg/dL, and serum creatinine is 3.1 mg/dL. Which of the following is the most likely underlying cause for her findings? □ (A) Defective ADP-induced platelet aggregation □ (B) Presence of antibodies against von Willebrand's factor metalloproteinase □ (C) Formation of auto-antibodies to platelet glycoproteins IIb/IIIa and Ib-IX □ (D) Circulating toxin that injures capillary endothelium □ (E) Inappropriate release of thromboplastic substances into blood □ (F) Decreased factor VIII activity

Presence of antibodies against von Willebrand's factor metalloproteinase

A 69-year-old man notices the presence of "lumps" in the right side of his neck that have been enlarging over the past year. Physical examination shows firm, nontender posterior cervical lymph nodes 1 to 2 cm in diameter. The overlying skin is intact and not erythematous. A lymph node is biopsied. Which of the following histologic features provides the best evidence for malignant lymphoma in this node? □ (A) Presence of lymphoid cells positive for kappa, but not lambda, light chains □ (B) Absence of a pattern of follicles with germinal centers □ (C) Proliferation of small capillaries in the medullary and paracortical regions □ (D) Presence of cells that stain with monoclonal antibody to the CD30 antigen □ (E) Absence of plasma cells and immunoblasts in sinusoidal spaces

Presence of lymphoid cells positive for kappa, but not lambda, light chains

A 15-year-old boy has developed a cough and a high fever over the past 4 days. On physical examination, he has a temperature of 39.2°C. Diffuse rales are heard over all lung fields. Laboratory studies show hemoglobin, 14.8 g/dL; hematocrit, 44.4%; platelet count, 496,000/mm3; and WBC count, 15,600/mm3. Examination of the peripheral blood smear shows RBCs with marked anisocytosis and Howell-Jolly bodies. A sputum culture grows Haemophilus influenzae. Which of the following is the most likely diagnosis? □ (A) DiGeorge syndrome □ (B) Galactosemia □ (C) Gaucher disease □ (D) Myeloproliferative disorder □ (E) Prior splenectomy □ (F) Trisomy 21

Prior splenectomy

A 49-year-old man goes to his physician for a checkup and is found on physical examination to have a blood pressure of 160/110 mm Hg, but no other abnormalities. Laboratory studies show serum glucose of 75 mg/dL, creatinine of 1.3 mg/dL, and urea nitrogen of 20 mg/dL. His plasma renin is elevated. CT angiography shows marked stenosis of his renal arteries. He is treated with an angiotensin-converting enzyme inhibitor. A week later, he has a headache for which he takes ibuprofen. Over the next day, his urine output decreases. A reduction in which of the following chemical mediators most likely caused his reduced urine output? □ (A) Aldosterone □ (B) Histamine □ (C) Nitric oxide □ (D) Prostaglandin □ (E) Tumor necrosis factor

Prostaglandin

For the past year, a 65-year-old man has had multiple, recurrent urinary tract infections. Escherichia coli and streptococcal organisms have been cultured from his urine during several of these episodes, with bacterial counts of more than 105/mL. He has difficulty with urination, including starting and stopping the urinary stream. Over the past week, he has again developed burning pain with urination. Urinalysis shows a pH of 6.5, and specific gravity of 1.020. No blood or protein is present in the urine. Tests for leukocyte esterase and nitrite are positive. Microscopic examination of the urine shows numerous WBCs and a few WBC casts. Which of the following is the most likely diagnosis? □ (A) Neisseria gonorrhoeae infection □ (B) Prostatic nodular hyperplasia □ (C) Phimosis □ (D) Epispadias □ (E) Adenocarcinoma of the prostate gland □ (F) Vesicoureteral reflux

Prostatic nodular hyperplasia

A 45-year-old woman has experienced malaise with nausea and vomiting for 3 months. On physical examination, she has scleral icterus and a yellowish hue to her skin. She has difficulty remembering three objects after 3 minutes. There are no neurologic deficits. Laboratory studies show a positive serologic test result for hepatitis C, a serum ALT of 310 U/L, AST of 275 U/L, total bilirubin of 7.6 mg/dL, direct bilirubin of 5.8 mg/dL, alkaline phosphatase of 75 U/L, and ammonia of 55 μmol/L. An abnormal result of which of the following laboratory studies of hemostatic function is most likely to be reported? □ (A) Immunoassay for plasma von Willebrand factor □ (B) Platelet count □ (C) Prothrombin time □ (D) Fibrin split products □ (E) Platelet aggregation

Prothrombin time

A 28-year-old man is brought to the emergency department with shock that developed over the past 12 hours. On physical examination, his temperature is 38.6°C, pulse is 101/min, respirations are 19/min, and blood pressure is 80/40 mm Hg. Needle tracks are noted in the left antecubital fossa. Crackles are heard over the lower lung fields. CBC shows hemoglobin, 14.1 g/dL; hematocrit, 42.6%; MCV, 93 μm3; platelet count, 127,500/mm3; and WBC count, 12,150/mm3 with 71% segmented neutrophils, 8% bands, 14% lymphocytes, and 7% monocytes. The neutrophils show cytoplasmic toxic granulations and Döhle bodies. Which of the following is the most likely diagnosis? □ (A) Pulmonary Mycobacterium tuberculosis □ (B) Acute myelogenous leukemia □ (C) Chronic myelogenous leukemia □ (D) Pseudomonas aeruginosa septicemia □ (E) Infectious mononucleosis □ (F) Pneumocystis carinii pneumonia

Pseudomonas aeruginosa septicemia

A 17-year-old girl has had arthralgias and myalgias for several months. During the past week, she has noticed a decreased output of urine, which is reddish brown. On physical examination, her blood pressure is 160/100 mm Hg, and she has an erythematous malar skin rash. The ANA and anti-double-stranded DNA test results are positive. The serum urea nitrogen level is 52 mg/dL. Which of the following urinalysis findings is most likely to be reported for this patient? □ (A) Eosinophils □ (B) Glucose □ (C) Ketones □ (D) Myoglobin □ (E) Oval fat bodies □ (F) RBC casts □ (G) Triple phosphate crystals □ (H) Uric acid crystals □ (I) Waxy casts

RBC casts

Which of the following statements about Rocky Mountain Spotted Fever (RMSF) is false? (A) Rickettsia rickettsii causes a vasculitis syndrome by its ability to invade endothelial cells. (B) RMSF occurs only between April and October. (C) RMSF is often associated with hyponatremia. (D) The rash of RMSF spreads centripetally (extremities to trunk). (E) Antibodies to R. rickettsii are usually dateable in the first month after infection.

RMSF occurs only between April and October

A 47-year-old man has had a decreased urine output over the past 10 days. On physical examination, he is afebrile. Urinalysis shows 1+ proteinuria, 4+ hematuria, urobilinogen, and no glucose or ketones. Microscopic examination of the urine shows few WBCs and some RBCs with RBC casts. A renal biopsy is done, and the light microscopic appearance of a PAS-stained specimen is shown in the figure. What is the most likely clinical course in this patient? □ (A) Acute renal failure that is reversible with supportive therapy □ (B) Slowly developing renal failure that is unresponsive to corticosteroid treatment □ (C) Rapidly progressive renal failure accompanied by hemoptysis □ (D) Stable clinical course with intermittent hematuria □ (E) Fever, leukocytosis, and endotoxic shock

Rapidly progressive renal failure accompanied by hemoptysis

A 29-year-old woman has had malaise and a low-grade fever for the past week. On physical examination, she appears very pale. She has a history of chronic anemia, and spherocytes are observed on a peripheral blood smear. Her hematocrit, which normally ranges from 35% to 38%, is now 28%, and the reticulocyte count is very low. The serum bilirubin level is 0.9 mg/dL. Which of the following events is most likely to have occurred in this patient? □ (A) Development of anti-RBC antibodies □ (B) Disseminated intravascular coagulation □ (C) Accelerated extravascular hemolysis in the spleen □ (D) Reduced erythropoiesis from parvovirus infection □ (E) Superimposed iron deficiency

Reduced erythropoiesis from parvovirus infection

A 45-year-old man has experienced recurrent fevers and a 6-kg weight loss over the past 5 months. On physical examination, his temperature is 37.5°C, and he has cervical lymphadenopathy. The patient reports that the adenopathy becomes very tender after he drinks a six-pack of beer. A lymph node biopsy specimen shows effacement of the nodal architecture by a population of small lymphocytes, plasma cells, eosinophils, and macrophages. Which of the following additional cell types, which stains positively for CD15, is most likely to be found in this disease? □ (A) Reed-Sternberg cell □ (B) Immunoblast □ (C) Epithelioid cell □ (D) Neutrophils □ (E) Mast cell

Reed-Sternberg cell

A 65-year-old woman has experienced increasing malaise with nocturia and polyuria for the past year. On physical examination, her blood pressure is 170/95 mm Hg. Urinalysis shows a pH of 7.5; specific gravity 1.010; 1+ proteinuria; and no glucose, blood, or ketones. The tests for leukocyte esterase and nitrite yield positive results, and levels of serum urea nitrogen and serum creatinine are elevated. Her clinical course is characterized by worsening renal failure, and she dies of bronchopneumonia. At autopsy, the kidneys are shrunken but unequal in size, and have deep, irregular surface scars. On sectioning, the calyces underlying the cortical scars are blunted and deformed. What is the most likely cause of renal failure in this patient? □ (A) Chronic glomerulonephritis □ (B) Essential hypertension □ (C) Reflux nephropathy □ (D) Autosomal-dominant polycystic kidney disease □ (E) Systemic lupus erythematosus

Reflux nephropathy

For the past 20 years, a 69-year-old man with chronic arthritis has taken more than 3 g of analgesics per day, including phenacetin, aspirin, and acetaminophen. He sees his physician because of increasing malaise, nausea, and diminished mentation. On physical examination, his blood pressure is 156/92 mm Hg. Laboratory findings show serum urea nitrogen level of 68 mg/dL and creatinine level of 7.1 mg/dL. CBC shows hemoglobin, 11.7 g/dL; hematocrit, 35.1%; platelet count, 188,500/mm3; and WBC count, 5385/mm3. Which of the following renal diseases is this patient most likely to develop? □ (A) Hydronephrosis □ (B) Chronic glomerulonephritis □ (C) Renal papillary necrosis □ (D) Renal cell carcinoma □ (E) Acute tubular necrosis

Renal papillary necrosis

A 40 year old woman with a history of asthma presents with 4 hr of shortness of breath. On physical examination, she is wheezing audibly. On auscultation, she is moving air poorly. Arterial blood gas reveals pH 7.27 and pCO2 70. What is the primary acid base disorder? (A) Metabolic acidosis (B) Metabolic alkalosis (C) Respiratory acidosis (D) Respiratory alkalosis

Respiratory acidosis

A healthy 19-year-old woman suffered blunt abdominal trauma in a motor vehicle accident. On admission to the hospital, her initial hematocrit was 33%, but over the next hour, it decreased to 28%. A paracentesis yielded serosanguineous fluid. She was taken to surgery, where a liver laceration was repaired, and 1 L of bloody fluid was removed from the peritoneal cavity. She remained stable. A CBC performed 3 days later is most likely to show which of the following morphologic findings in RBCs in the peripheral blood? □ (A) Reticulocytosis □ (B) Leukoerythroblastosis □ (C) Basophilic stippling □ (D) Hypochromia □ (E) Schistocytes

Reticulocytosis

A 26-year-old man is involved in a motor vehicle accident and sustains acute blood loss. He is hypotensive for several hours before paramedical personnel arrive. They stabilize the bleeding and transport him to a hospital, where he receives a transfusion of 3 U of packed RBCs. Over the next week, the serum urea nitrogen level increases to 48 mg/dL, the serum creatinine level increases to 5 mg/dL, and the urine output decreases. He undergoes hemodialysis for the next 2 weeks and then develops marked polyuria, with urine output of 2 to 3 L/day. His recovery is complicated by bronchopneumonia, but renal function gradually returns to normal. The patient's transient renal disease is best characterized by which of the following histologic features? □ (A) Glomerular crescents in Bowman space □ (B) Interstitial lymphocytic infiltrates □ (C) Arteriolar fibrinoid necrosis □ (D) Nodular glomerulosclerosis □ (E) Rupture of tubular basement membrane

Rupture of tubular basement membrane

A 61-year-old woman sees the physician because she has experienced increasing malaise for the past 5 years. On physical examination, there are no abnormalities other than a blood pressure of 150/95 mm Hg. One week later, she dies suddenly. At autopsy, both kidneys have the external (left panel) and bisected (right panel) appearance as shown in the figure. Which of the following conditions was the most probable cause of death? □ (A) Metastatic Wilms tumor □ (B) Ruptured berry aneurysm □ (C) Acute tubular necrosis □ (D) Disseminated intravascular coagulation □ (E) Pneumothorax

Ruptured berry aneurysm

A 60-year-old man has developed widespread ecchymoses over the skin in the past month. His medical history includes a diagnosis of mucinous adenocarcinoma of the rectum. On physical examination, he appears cachectic and pale. An abdominal CT scan shows multiple hepatic masses. Laboratory studies show prothrombin time of 30 seconds, partial thromboplastin time of 55 seconds, platelet count of 15,200/mm3, and fibrinogen level of 75 mg/dL, and fibrin split product levels (D dimer) that are very elevated. Which of the following morphologic findings is most likely to be present on examination of his peripheral blood smear? □ (A) Howell-Jolly bodies □ (B) Teardrop cells □ (C) Macro-ovalocytes □ (D) Schistocytes □ (E) Target cells

Schistocytes

A 29 year old man complains of a vague feeling of heaviness in his scrotum, but his pain has not increased for the past five months. He is otherwise healthy. Physical examination reveals that the right testis is slightly larger than the left testis. An ultrasound scan reveals the presence of a solid, circumscribed, 1.5-cm mass in the body of the right testis. The representative gross appearance of the mass is shown below. A biopsy is performed, and microscopic examination of the mass shows uniform nests of cells with distinct cell borders, glycogen-rich cytoplasm, and round nuclei with prominent nucleoli. There are aggregates of lymphocytes between these nests of cells. The microscopic appearance is also illustrated below. Which of the following is the most likely diagnosis? (A) Choriocarcinoma (B) Teratoma (C) Seminoma (D) Lymphoma (E) Embryonal carcinoma

Seminoma

A 42-year-old man has had right flank pain for the past 2 days. On physical examination, his temperature is 37.4°C, pulse is 70/min, respirations are 14/min, and blood pressure is 130/85 mm Hg. Laboratory studies show a serum creatinine level of 1.1 mg/dL. Urinalysis shows no blood, protein, or glucose, and microscopic examination of the urine shows no WBCs or RBCs. Abdominal CT scan shows a 7-cm eccentric lesion of the upper pole of the right kidney. The lesion is well circumscribed and cystic with a thin wall and focal hemorrhage. What is the most likely diagnosis? □ (A) Acute pyelonephritis □ (B) Acute tubular necrosis □ (C) Diabetic nephropathy □ (D) Hydronephrosis □ (E) Simple renal cyst □ (F) Rapidly progressive glomerulonephritis □ (G) Renal cell carcinoma □ (H) Urothelial carcinoma

Simple renal cyst

A 39-year-old woman felt a lump in her breast 1 week ago. She visits the physician, who palpates a firm, fixed, irregular 3-cm mass in the upper outer quadrant of the right breast and a firm, nontender lymph node in the right axilla. A lumpectomy and axillary node dissection are performed, and microscopic examination shows an infiltrating ductal carcinoma present in the breast. Flow cytometric analysis of the node shows a polyclonal population of CD3+, CD19+, CD20+, and CD68+ cells with no aneuploidy or increase in S phase. Which of the following is most likely to be present on microscopic examination of this axillary node? □ (A) Acute lymphadenitis □ (B) Diffuse large B-cell lymphoma □ (C) Metastatic infiltrating ductal carcinoma □ (D) Necrotizing granulomas □ (E) Plasmacytosis □ (F) Sinus histiocytosis

Sinus histiocytosis

A 45 year old oilman from the US spends two years in Nigeria. He returns with chronic pruritus (itching). During his interview, he says the scratching is "maddening". The most appropriate diagnostic test for him is (A) Bone marrow biopsy (B) Blood culture (C) Skin snip (D) Intraperitoneal injection of a guinea pig with the patient's blood

Skin snip

A 62-year-old man has had several episodes of hematuria over the past week. He has not experienced increased urinary frequency or dysuria. On physical examination, there are no remarkable findings. Urinalysis shows 4+ hematuria. The urine culture is negative. A cystoscopy is performed, and a 2-cm sessile, friable mass is seen on the right bladder wall. A biopsy specimen is obtained; the microscopic appearance is shown in the figure. Which of the following risk factors is most important in the pathogenesis of this bladder lesion? □ (A) Smoking cigarettes □ (B) Schistosomiasis □ (C) Diabetes mellitus □ (D) Chronic bacterial cystitis □ (E) Nodular prostatic hyperplasia

Smoking cigarettes

For the past 6 months, a 72-year-old woman has noticed a slowly enlarging mass on the urethra. The mass causes local pain and irritation and is now bleeding. Physical examination shows a 2.5-cm warty, ulcerated mass protruding from the external urethral meatus. There are no lesions on the labia or vagina. A biopsy specimen of the lesion is most likely to identify which of the following? □ (A) Embryonal rhabdomyosarcoma □ (B) Leiomyoma □ (C) Papilloma □ (D) Squamous cell carcinoma □ (E) Syphilitic chancre

Squamous cell carcinoma

A 77-year-old man has had increasing difficulties with urination for the past 2 years. He has difficulty starting and stopping the urine stream. On physical examination, his temperature is 37°C, and blood pressure is 130/85 mm Hg. The figure shows the representative gross appearance of the bladder. Which of the following laboratory findings is most likely to be reported in this patient? □ (A) Positive ANA test result □ (B) Urine culture positive for Mycobacterium tuberculosis □ (C) Hemoglobin concentration of 22.5 g/dL □ (D) Schistosoma haematobium eggs in urine □ (E) Stable prostate-specific antigen level of 5 ng/mL

Stable prostate-specific antigen level of 5 ng/mL

An 85-year-old man comes to the physician because he had experienced urinary hesitancy and nocturia for the past year. He has had increasing back pain for the past 6 months. On digital rectal examination, there is a hard, irregular prostate gland. A bone scan shows increased areas of uptake in the thoracic and lumbar vertebrae. Laboratory studies show a serum alkaline phosphatase level of 300 U/L, serum prostatic acid phosphatase level of 8 ng/mL, and serum prostate-specific antigen (PSA) level of 72 ng/mL. The blood urea nitrogen concentration is 44 mg/dL, and the serum creatinine level is 3.8 mg/dL. Transrectal biopsy specimens of all lobes of the prostate are obtained. Microscopic examination shows that more than 90% of the tissue has a pattern of cords and sheets of cells with hyperchromatic pleomorphic nuclei, prominent nuclei, and scant cytoplasm. Which of the following is the best classification for this patient's disease? (Stage/Gleason Grade) A. A1/1,1 B. A2/1,2 C. B1/2,3 D. B2/3,3 E. C1/3,4 F. C2/4,4 G. D1/4,5 H. D2/5,5

Stage: D2 Gleason grade: 5,5

A 51-year-old woman has had recurrent urinary tract infections for the past 15 years. On many of these occasions, Proteus mirabilis was cultured from her urine. For the past 4 days, she has had a burning pain on urination and urinary frequency. On physical examination, her temperature is 37.9°C, pulse is 70/min, respirations are 15/min, and blood pressure is 135/85 mm Hg. There is marked tenderness on deep pressure over the right costovertebral angle and on deep abdominal palpation. Urinalysis shows a pH of 7.5; specific gravity 1.020; 1+ hematuria; and no protein, glucose, or ketones. Microscopic examination of the urine shows many RBCs, WBCs, and triple-phosphate crystals. Which of the following renal lesions is most likely to be present? □ (A) Renal cell carcinoma □ (B) Acute tubular necrosis □ (C) Malignant nephrosclerosis □ (D) Staghorn calculus □ (E) Papillary necrosis

Staghorn calculus

A 28-year-old, previously healthy man has noted increasing fatigue for the past 6 months and formation of bruises after minimal trauma. Over the past 2 days, he has developed a cough. On physical examination, his temperature is 38.9°C, and he has diffuse rales in both lungs. He has no hepatosplenomegaly and no lymphadenopathy. Laboratory findings include a sputum culture positive for Streptococcus pneumoniae, hemoglobin of 7.2 g/dL, hematocrit of 21.7%, platelet count of 23,400/mm3, WBC count of 1310/mm3, prothrombin time of 13 seconds, partial thromboplastin time of 28 seconds, and total bilirubin of 1 mg/dL. The ANA test result is negative. What is the most likely explanation of these findings? □ (A) Hemolysis of antibody-coated cells □ (B) Hypersplenism □ (C) Increased susceptibility to lysis by complement □ (D) Metastatic adenocarcinoma □ (E) Nuclear maturation defects resulting from impaired DNA synthesis □ (F) Stem cell defect □ (G) Varicella-zoster virus infection

Stem cell defect

A 23-year-old man undergoing chemotherapy for acute lymphoblastic leukemia has developed a fever and abdominal pain within the past week. He now has a severe cough. On physical examination, his temperature is 38.4°C. On auscultation, crackles are heard over all lung fields. Laboratory studies show hemoglobin, 12.8 g/dL; hematocrit, 39%; MCV, 90 μm3; platelet count, 221,000/mm3; and WBC count, 16,475/mm3 with 51% segmented neutrophils, 5% bands, 18% lymphocytes, 8% monocytes, and 18% eosinophils. Infection with which of the following organisms is most likely to be complicating the course of this patient's disease? □ (A) Cryptococcus neoformans □ (B) Cytomegalovirus □ (C) Helicobacter pylori □ (D) Hepatitis C virus □ (E) Pseudomonas aeruginosa □ (F) Strongyloides stercoralis □ (G) Toxoplasma gondii □ (H) Varicella-zoster virus

Strongyloides stercoralis

A 20-year-old woman, G1, P0, who is in the third trimester, has felt minimal fetal movement. An ultrasound scan shows a markedly decreased amniotic fluid index characteristic of oligohydramnios. She gives birth to a stillborn male fetus at 33 weeks' gestation. At autopsy, there are deformations resulting from marked oligohydramnios, including flattening of the facies, varus deformities of the feet, and marked pulmonary hypoplasia. Microscopic examination of the liver shows multiple epithelium-lined cysts and a proliferation of bile ducts. Which of the following best describes the appearance of the kidneys in this fetus? □ (A) Bilaterally enlarged kidneys replaced by 1- to 4-cm, fluid-filled cysts □ (B) Bilaterally shrunken kidneys with uniformly finely granular cortical surfaces □ (C) Decreased overall size of the right kidney and normal-sized left kidney □ (D) Irregular cortical scars in asymmetrically shrunken kidneys with marked calyceal dilation □ (E) Marked bilateral renal pelvic and calyceal dilation with thinning of the cortices □ (F) Normal-sized kidneys with smooth cortical surfaces □ (G) Symmetrically enlarged kidneys composed of small, radially arranged cysts

Symmetrically enlarged kidneys composed of small, radially arranged cysts

A 30-year-old woman reports becoming increasingly tired for the past 5 months. On physical examination, she is afebrile and has mild splenomegaly. Laboratory studies show a hemoglobin concentration of 11.8 g/dL and hematocrit of 35.1%. The peripheral blood smear shows spherocytes and rare nucleated RBCs. Direct and indirect Coombs test results are positive at 37°C, although not at 4°C. Which of the following underlying diseases is most likely to be diagnosed in this patient? □ (A) Infectious mononucleosis □ (B) Mycoplasma pneumoniae infection □ (C) Hereditary spherocytosis □ (D) Escherichia coli septicemia □ (E) Systemic lupus erythematosus

Systemic lupus erythematosus

Which of the following statements regarding anemias is false? (A) Many symptoms associated with chronic renal failure are caused by anemia. (B) Thalassemia can cause a microcytic anemia, even when iron levels are normal. (C) Lack of gastric acid reduces absorption of Vitamin B12 and can cause a macrocytic anemia. (D) Renal hypoxia caused by anemia is a major stimulus for the biosynthesis of erythropoietin. (E) The most common cause of moderate-to-severe iron deficiency anemia in the US is poor diet.

The most common cause of moderate-to-severe iron deficiency anemia in the US is poor diet

A 33-year-old man has noted asymmetric enlargement of the scrotum over the past 4 months. On physical examination, the right testis is twice its normal size and has increased tenderness to palpation. The right testis is removed. The epididymis and the upper aspect of the right testis have extensive granulomatous inflammation with epithelioid cells, Langhans giant cells, and caseous necrosis. Which of the following is the most likely cause of these findings? □ (A) Mumps □ (B) Syphilis □ (C) Tuberculosis □ (D) Gonorrhea □ (E) Sarcoidosis

Tuberculosis

A 45 year man is found to have enlarged lymph nodes in his neck, Ann Arbor Stage I. He noted them for the first time 12 months ago, but they got smaller and sometimes they became large again. As a consequence, he felt they were not significant. Laboratory studies reveal Hct 38%, Hgb 13 g/dl, WBC count 8250/mm3, and platelet count 187,000/mm3. LDH is normal. A lymph node is biopsied and the result is shown below. What should you tell him about his prognosis? (A) Your life expectancy with chemotherapy and radiation is less than one year. (B) Disease progress can be slowed, but not cured, with Lenalidomide and Stem Cell Transplant. (C) Tumor lysis and disseminated intravascular coagulation will cause death rapidly. (D) This disease will transform into acute myelogenous leukemia within two years. (E) This is an indolent disease with a 70% 5 year survival.

This is an indolent disease with a 70% 5 year survival

A 72-year-old man with Alzheimer disease dies of pneumonia. The gross appearance of the right kidney at autopsy is shown in the figure. The left kidney is normal in size, with a smooth cortical surface and a single 0.6-cm, fluid-filled cyst. The appearance of the right kidney is most suggestive of renal injury from which of the following? □ (A) Ureteral obstruction □ (B) Benign nephrosclerosis □ (C) Analgesic abuse □ (D) Chronic pyelonephritis □ (E) Diabetes mellitus

Ureteral obstruction

A 28-year-old man is diagnosed with acute myelogenous leukemia (M2). After induction with a multiagent chemotherapy protocol, he has an episode of lower abdominal pain accompanied by passage of red-colored urine. He has no fever, dysuria, or urinary frequency. On physical examination, there are no remarkable findings. Urinalysis shows a pH of 5.5; specific gravity 1.021; 2+ hematuria; and no protein, ketones, or glucose. There are no remarkable findings on an abdominal radiograph. Which of the following additional urinalysis findings is most likely to be reported for this patient? □ (A) Bence Jones protein □ (B) Eosinophils □ (C) Myoglobin □ (D) Oval fat bodies □ (E) RBC casts □ (F) Triple phosphate crystals □ (G) Uric acid crystals □ (H) Waxy casts □ (I) WBC casts

Uric acid crystals

A 54-year-old woman sees her physician because of sudden onset of headaches and photophobia. This condition has been worsening for the past 2 days. On physical examination, she has a temperature of 38°C and is disoriented. CBC shows hemoglobin of 11.2 g/dL, hematocrit of 33.7%, MCV of 94 μm3, platelet count of 32,000/mm3, and WBC count of 9900/mm3. The peripheral blood smear shows schistocytes. The serum urea nitrogen level is 38 mg/dL, and the creatinine level is 3.9 mg/dL. Which of the following is the most likely diagnosis? □ (A) β-Thalassemia major □ (B) Disseminated intravascular coagulation □ (C) Hereditary spherocytosis □ (D) Idiopathic thrombocytopenic purpura □ (E) Paroxysmal nocturnal hemoglobinuria □ (F) Thrombotic thrombocytopenic purpura □ (G) Warm autoimmune hemolytic anemia

Thrombotic thrombocytopenic purpura

A clinical study is performed involving adult patients diagnosed with microangiopathic hemolytic anemia. The patients did not have diarrhea. The patients had schistocytes present on peripheral blood smears. Some of these patients were found to have a deficiency of a metalloproteinase known as ADAMTS-13. Which of the following conditions were these ADAMTS-13-deficient patients most likely to have? □ (A) Disseminated intravascular coagulopathy (DIC) □ (B) Hemolytic-uremic syndrome (HUS) □ (C) Heparin-induced thrombocytopenia (HIT) □ (D) Idiopathic thrombocytopenic purpura (ITP) □ (E) Systemic lupus erythematosus (SLE) □ (F) Thrombotic thrombocytopenic purpura (TTP)

Thrombotic thrombocytopenic purpura (TTP)

A 20-year-old man is left at the door of the emergency department by his "friends" after they spent an evening at a local pub. On examination, his vital signs are temperature, 37°C; pulse, 110/min; respirations, 20/min; and blood pressure, 75/40 mm Hg. He has left upper quadrant tenderness on palpation. An abdominal CT scan was obtained and is shown in the figure. What is the most likely etiology for this man's findings? □ (A) Amyloidosis □ (B) Cirrhosis □ (C) Gaucher disease □ (D) Myeloproliferative disorder □ (E) Nonbacterial thrombotic endocarditis □ (F) Salmonella typhi infection □ (G) Trauma

Trauma

A 65-year-old man recently retired after many years in a job that involved exposure to aniline dyes, including ßnaphthylamine. One month ago, he had an episode of hematuria that was not accompanied by abdominal pain. On physical examination, there are no abnormal findings. Urinalysis shows 4+ hematuria, and no ketones, glucose, or protein. Microscopic examination of the urine shows RBCs that are too numerous to count, 5 to 10 WBCs per high-power field, and no crystals or casts. The result of a urine culture is negative. What is the most likely diagnosis? □ (A) Renal cell carcinoma □ (B) Hemorrhagic cystitis □ (C) Tubercular cystitis □ (D) Urothelial carcinoma □ (E) Squamous cell carcinoma of the urethra

Urothelial carcinoma

A 19-year-old woman has had a fever and chills accompanied by right flank pain for the past 3 days. On physical examination, her temperature is 38.3°C, her blood pressure is 150/90 mm Hg, and there is right costovertebral angle tenderness. Laboratory findings show a serum glucose level of 77 mg/dL and creatinine level of 1 mg/dL. Urinalysis shows a pH of 6.5; specific gravity 1.018; and no protein, blood, glucose, or ketones. Microscopic examination of the urine shows many WBCs and WBC casts. Which of the following factors is most important in the pathogenesis of the renal disease affecting this patient? □ (A) Age □ (B) Sex □ (C) Vesicoureteral reflux □ (D) Blood pressure □ (E) Focus of infection in the lungs

Vesicoureteral reflux

A 30-year-old woman with a history of recurrent urinary tract infections has had a high fever for the past 3 days. On physical examination, her temperature is 38.4°C. There is marked abdominal tenderness on deep palpation. A renal ultrasound scan shows an enlarged right kidney with pelvic and calyceal enlargement and cortical thinning; the left kidney appears normal. A right nephrectomy is done, and microscopic examination shows inflammatory infiltrates extending from the medulla to the cortex, with tubular destruction and extensive interstitial fibrosis. Lymphocytes, plasma cells, and neutrophils are abundant. Which of the following is most likely to produce these findings? □ (A) Benign nephrosclerosis □ (B) Vesicoureteral reflux □ (C) Lupus nephritis □ (D) Systemic amyloidosis □ (E) Congestive heart failure □ (F) Autosomal-dominant polycystic kidney disease

Vesicoureteral reflux

After a 6 month worldwide trip that included travel to Asia and Africa, a previously healthy 25 year old graduate student presents with fever, anorexia, and weight loss. Your examination is significant for splenomegaly, hepatomegaly, lymphadenopathy, and pallor. Laboratory results reflect anemia, leucopenia, and thrombocytopenia. You suspect (A) Cutaneous leismaniasis (B) Mucosal leishmaniasis (C) Visceral leishmaniasis (D) Filariasis (E) Trypanosomiasis

Visceral leishmaniasis

A 39-year-old woman sees her physician because she has experienced abdominal pain and intermittent low-volume diarrhea for the past 3 months. On physical examination, she is afebrile. A stool sample is positive for occult blood. A colonoscopy is performed, and biopsy specimens from the terminal ileum and colon show microscopic findings consistent with Crohn's disease. Because she has failed to respond to medical therapy, surgery is warranted, and part of the colon and terminal ileum are removed. She is transfused with 2 U of packed RBCs during surgery. Several weeks later, she appears healthy, but complains of easy fatigability. On investigation, CBC findings show hemoglobin of 10.6 g/dL, hematocrit of 31.6%, RBC count of 2.69 million/μL, MCV of 118 μm3, platelet count of 378,000/mm3, and WBC count of 9800/mm3. The reticulocyte count is 0.3%. Which of the following is most likely to produce these findings? □ (A) Hemolytic anemia □ (B) Aplastic anemia □ (C) Chronic blood loss □ (D) Vitamin B12 deficiency □ (E) Anemia of chronic disease □ (F) Bone marrow metastases

Vitamin B12 deficiency

A 76-year-old woman notices that small, pinpoint-to-blotchy areas of superficial hemorrhage have appeared on her gums and on the skin of her arms and legs over the past 3 weeks. On physical examination, she is afebrile and has no organomegaly. Laboratory studies show a normal prothrombin time and partial thromboplastin time. CBC shows hemoglobin of 12.7 g/dL, hematocrit of 37.2%, MCV of 80 μm3, platelet count of 276,000/mm3, and WBC count of 5600/mm3. Platelet function studies and fibrinogen level are normal, and no fibrin split products are detectable. Which of the following conditions best explains these findings? □ (A) Macronodular cirrhosis □ (B) Chronic renal failure □ (C) Meningococcemia □ (D) Vitamin C deficiency □ (E) Metastatic carcinoma

Vitamin C deficiency

A 32-year-old man with a history of intravenous drug use comes to the emergency department because he has had a high fever for the past 2 days. On physical examination, his temperature is 38.4°C. He has a palpable spleen tip, bilateral costovertebral angle tenderness, and diastolic cardiac murmur. Laboratory findings show a serum urea nitrogen level of 15 mg/dL. Urinalysis shows 2+ hematuria, and no glucose, protein, or ketones. A blood culture is positive for Staphylococcus aureus. Which of the following best describes the kidneys in this patient? □ (A) Bilaterally enlarged kidneys replaced by 1- to 4-cm, fluid-filled cysts □ (B) Bilaterally shrunken kidneys with uniformly finely granular cortical surfaces □ (C) Irregular cortical scars in asymmetrically shrunken kidneys with marked calyceal dilation □ (D) Marked bilateral renal pelvic and calyceal dilation with thinning of the cortices □ (E) Normal-sized kidneys with smooth cortical surfaces □ (F) Scattered petechial hemorrhages in slightly swollen kidneys □ (G) Wedge-shaped regions of yellow-white cortical necrosis involving both kidneys

Wedge-shaped regions of yellow-white cortical necrosis involving both kidneys

A 44-year-old man has developed a fever, nonproductive cough, and decreased urine output over the past 3 days. On physical examination, his temperature is 37.7°C, and blood pressure is 145/95 mm Hg. He has sinusitis. On auscultation, crackles are heard over all lung fields. A chest radiograph shows bilateral patchy infiltrates and nodules. The serum creatinine level is 4.1 mg/dL, and the urea nitrogen level is 43 mg/dL. The results of serologic testing are negative for ANA, but positive for C-ANCA. A renal biopsy specimen shows glomerular crescents and granulomatous vasculitis. The result of immunofluorescence staining with anti-IgG and anti-C3 antibodies is negative. What is the most likely diagnosis? □ (A) Focal segmental glomerulosclerosis □ (B) Goodpasture syndrome □ (C) Lupus nephritis □ (D) Membranoproliferative glomerulonephritis type II □ (E) Membranous glomerulonephritis □ (F) Postinfectious glomerulonephritis □ (G) Wegener's granulomatosis

Wegener's granulomatosis

A 4-year-old girl has complained of abdominal pain for the past month. On physical examination, she is febrile, and palpation of the abdomen shows a tender mass on the right. Bowel sounds are present. Laboratory studies show hematuria without proteinuria. Abdominal CT scan shows a 12-cm, circumscribed, solid mass in the right kidney. A right nephrectomy is done; the gross appearance of the mass is shown in the figure. What is the most likely diagnosis? □ (A) Angiomyolipoma □ (B) Interstitial cell tumor □ (C) Renal cell carcinoma □ (D) Transitional cell carcinoma □ (E) Wilms tumor, nephroblastoma

Wilms tumor, nephroblastoma

A 32-year-old man has noticed an increased feeling of heaviness in his scrotum for the past 10 months. On physical examination, the left testis is three times the size of the right testis and is firm on palpation. An ultrasound scan shows a 6-cm solid mass within the body of the left testis. Laboratory studies include an elevated serum α-fetoprotein level. Which of the following cellular components is most likely to be present in this mass? □ (A) Yolk sac cells □ (B) Leydig cells □ (C) Seminoma cells □ (D) Cytotrophoblasts □ (E) Embryonal carcinoma cells □ (F) Lymphoblasts

Yolk sac cells

The mother of a 2-year-old boy notices that he has had increasing asymmetric enlargement of the scrotum over the past 6 months. On physical examination, there is a well-circumscribed, 2.5-cm mass in the left testis. A left orchiectomy is performed, and histologic examination of this mass shows sheets of cells and ill-defined glands composed of cuboidal cells, some of which contain eosinophilic hyaline globules. Microcysts and primitive glomeruloid structures also are seen. Immunohistochemical staining shows α-fetoprotein in the cytoplasm of the neoplastic cells. What is the most likely diagnosis? □ (A) Choriocarcinoma □ (B) Seminoma □ (C) Yolk sac tumor □ (D) Teratoma □ (E) Leydig cell tumor

Yolk sac tumor

A 19-year-old man comes to his physician for a routine health maintenance examination. On physical examination, there is no left testis palpable in the scrotum. The patient is healthy, has had no major illnesses, and has normal sexual function. In counseling this patient, which of the following statements regarding his condition would be most appropriate? □ (A) You will be unable to father children □ (B) You are at increased risk of developing a testicular tumor □ (C) This is a common finding in more than half of all men □ (D) This is an outcome of childhood mumps infection □ (E) This is an inherited disorder

You are at increased risk of developing a testicular tumor

A 23-year-old woman has had a history of bleeding problems all of her life, primarily heavy menstruation and bleeding gums. A sister and an uncle also have bleeding problems. Physical examination shows several bruises ranging in color from red to blue to purple on her arms and legs. There is no organomegaly, and no deformities are noted. Laboratory studies show hemoglobin, 9.5 g/dL; hematocrit, 28.2%; platelet count, 229,300/mm3; WBC count, 7185/mm3; prothrombin time, 12 seconds; and partial thromboplastin time, 38 seconds. A 1 : 1 dilution of the patient's plasma with normal pooled plasma corrects the partial thromboplastin time. Ristocetin-dependent platelet aggregation in patient plasma is markedly reduced. Factor VIII activity is 30% (reference range 50% to 150%). Which of the following responses should the physician use when advising the patient of potential consequences of this disease? □ (A) You might need allogeneic bone marrow transplantation □ (B) Expect increasing difficulty with joint mobility □ (C) Anticoagulation is needed to prevent deep venous thrombosis □ (D) You could experience excessive bleeding after oral surgery □ (E) A splenectomy might be necessary to control the disease

You could experience excessive bleeding after oral surgery

A 32-year-old man has reported easy fatigability since childhood. On physical examination, he is normally developed and is afebrile. Laboratory studies show hemoglobin of 8.8 g/dL, hematocrit of 26.3%, platelet count of 199,000/mm3, and WBC count of 5350/mm3. α-Globin inclusions are present in erythroblasts and erythrocytes, leading to increased phagocytosis by cells of the mononuclear phagocyte system. Hemoglobin electrophoresis shows hemoglobin A2 of 6%, hemoglobin F of 1%, and hemoglobin A1 of 93%. The serum ferritin level is 3090 ng/mL. Which of the following is the most likely diagnosis? □ (A) Autoimmune hemolytic anemia □ (B) Glucose-6-phosphate dehydrogenase deficiency □ (C) Megaloblastic anemia □ (D) β-Thalassemia □ (E) Sickle cell anemia □ (F) Paroxysmal nocturnal hemoglobinuria

β-Thalassemia

A 25-year-old woman has a 3-year history of arthralgias. Physical examination shows no joint deformity, but she appears pale. Laboratory studies show total RBC count of 4.7 million/mm3, hemoglobin of 12.5 g/dL, hematocrit of 37.1%, platelet count of 217,000/mm3, and WBC count of 5890/mm3. The peripheral blood smear shows hypochromic and microcytic RBCs. Total serum iron and ferritin levels are normal. Hemoglobin electrophoresis shows an elevated hemoglobin A2 level of about 5.8%. What is the most likely diagnosis? □ (A) Autoimmune hemolytic anemia □ (B) β-Thalassemia minor □ (C) Infection with Plasmodium vivax □ (D) Anemia of chronic disease □ (E) Iron deficiency anemia

β-Thalassemia minor


Kaugnay na mga set ng pag-aaral

Medical Interventions Antibiotics

View Set

Geometry: Angle Theorems, Geometry - test 3

View Set

biology 101 chapter 1 homework questions

View Set

MAN 3320 CHAPTER 13 EMPLOYEE STAFFING AND BENEFITS

View Set

Business finance HW 1,2,3,4&5 Business Finance test/Final Questions

View Set

Acct 324 exam 3 ch.33, 34,35,42,43

View Set

All in ("ACCT 3326 Exam 3 (FINAL) CH 8 quiz Qs") folder combined

View Set

Karch Chapter 14: Bonus Antineoplastic

View Set

psych / human development > Chapter 6

View Set